july 2018 pennsylvania bar examination · july 2018 pennsylvania bar examination . essay questions...

82
JULY 2018 PENNSYLVANIA BAR EXAMINATION Essay Questions and Examiners’ Analyses and Performance Test Pennsylvania Board of Law Examiners 601 Commonwealth Avenue, Suite 3600 P.O. Box 62535 Harrisburg, PA 17106-2535 (717) 231-3350 www.pabarexam.org ©2018 Pennsylvania Board of Law Examiners

Upload: others

Post on 26-Apr-2020

11 views

Category:

Documents


0 download

TRANSCRIPT

Page 1: JULY 2018 PENNSYLVANIA BAR EXAMINATION · JULY 2018 PENNSYLVANIA BAR EXAMINATION . Essay Questions and Examiners’ Analyses. and . Performance Test . Pennsylvania Board of Law Examiners

JULY 2018

PENNSYLVANIA BAR

EXAMINATION

Essay Questions and Examiners’ Analyses

and

Performance Test

Pennsylvania Board of Law Examiners

601 Commonwealth Avenue, Suite 3600

P.O. Box 62535

Harrisburg, PA 17106-2535

(717) 231-3350

www.pabarexam.org

©2018 Pennsylvania Board of Law Examiners

Page 2: JULY 2018 PENNSYLVANIA BAR EXAMINATION · JULY 2018 PENNSYLVANIA BAR EXAMINATION . Essay Questions and Examiners’ Analyses. and . Performance Test . Pennsylvania Board of Law Examiners

i

Table of Contents

Index ..................................................................................................................................................ii

Question No. 1: Facts and Interrogatories, Examiner's Analysis and Grading Guidelines ..............1

Question No. 2: Facts and Interrogatories, Examiner's Analysis and Grading Guidelines ..............8

Question No. 3: Facts and Interrogatories, Examiner's Analysis and Grading Guidelines ..............16

Question No. 4: Facts and Interrogatories, Examiner's Analysis and Grading Guidelines ..............24

Question No. 5: Facts and Interrogatories, Examiner's Analysis and Grading Guidelines ..............35

Question No. 6: Facts and Interrogatories, Examiner's Analysis and Grading Guidelines ..............46

Performance Test and Grading Guidelines ........................................................................................55

Page 3: JULY 2018 PENNSYLVANIA BAR EXAMINATION · JULY 2018 PENNSYLVANIA BAR EXAMINATION . Essay Questions and Examiners’ Analyses. and . Performance Test . Pennsylvania Board of Law Examiners

Index

Question No. 1

1. Decedent’s Estates: undue influence

2. Decedent’s Estates: precatory words

3. Federal Income Tax: gambling income

4. Professional Responsibility: communication with represented person

Question No. 2

1. Business Organization: promoter liability, piercing corporate veil

2. U.C.C. Article II: limitation on remedies repair and replace

3. U.C.C. Article II: limitation on remedies consequential damages

Question No. 3

1. Criminal Law: involuntary manslaughter

2. Evidence: prior bad acts

3. Criminal Procedure: inventory search

4. Family Law: divorce

Question No. 4

1 Employment Law: disparate impact

2. Civil Procedure: class certification

3. Constitutional Law: mootness

ii

Page 4: JULY 2018 PENNSYLVANIA BAR EXAMINATION · JULY 2018 PENNSYLVANIA BAR EXAMINATION . Essay Questions and Examiners’ Analyses. and . Performance Test . Pennsylvania Board of Law Examiners

Question No. 5

1. Contracts: formation, counter-offer

2. Property: tenancy in common, joint tenancy with right of survivorship

3. Contracts: mitigation of damages

4. Property: equitable servitude

Question No. 6

1. Torts: professional negligence, vicarious liability

2. Civil Procedure: statute of limitations

3. Evidence: subsequent remedial measures

4. Torts: comparative negligence

iii

Page 5: JULY 2018 PENNSYLVANIA BAR EXAMINATION · JULY 2018 PENNSYLVANIA BAR EXAMINATION . Essay Questions and Examiners’ Analyses. and . Performance Test . Pennsylvania Board of Law Examiners

Question No. 1

Mike, a resident of B County, Pennsylvania, is 65 years old and single. Mike and his son,

Joe, were avid fans of the Pennsylvania Dogs professional football team. During football season,

they attended Dogs tailgate parties and games on Sundays, often drinking alcohol throughout the

day. Neither Mike nor Joe was observant of any religious denomination, and Mike was fond of

saying “football is our religion.” In addition to watching games, Mike routinely placed large

bets on the Dogs and other football teams, and he had net winnings of $40,000 from gambling on

football games during January 2018. On February 1, 2018, Mike placed a bet of $10,000 that the

Dogs would win their final game of the season. The Dogs lost 35-0, and Mike lost his $10,000

wager.

On February 5, Mike was diagnosed with stomach cancer and advised that he had only a

few months to live. In March, Mike met Robin, the pastor at the New Pennsylvania Church

(“Church”). Robin invited him to attend services that Sunday, which he did. From that day on,

Mike attended services at Church every week, and met with Robin twice a week for counseling.

Joe observed that his father remained cheerful and organized in his daily life, and Mike often

expressed his gratitude to Robin for helping him find peace.

Mike met with Lisa, an attorney and member of Church who also happened to be a friend

of Joe’s. Lisa prepared a new will, which specifically revoked Mike’s old will, which had

provided for his entire estate to pass to Joe. The new will read, in pertinent part, as follows:

I give the entirety of my estate as follows: one-half to Robin, and one-half to my

son Joe, to be used as Joe chooses. It is my wish that Joe stop drinking and attending

football games on Sundays and begin attending church and living a holy life. If

Robin or Joe predeceases me, his or her share shall pass to New Pennsylvania

Church. I name Robin as executor.

1

Page 6: JULY 2018 PENNSYLVANIA BAR EXAMINATION · JULY 2018 PENNSYLVANIA BAR EXAMINATION . Essay Questions and Examiners’ Analyses. and . Performance Test . Pennsylvania Board of Law Examiners

Mike properly executed the will on April 20, 2018, and properly signed a general durable power

of attorney naming Robin as his agent. On May 1, 2018, Mike died. Robin probated Mike’s will

on May 10, 2018.

Joe retained Andy, an attorney, to file a challenge to the will on the basis of undue

influence. Andy timely filed Joe’s claim, and a hearing date of August 15, 2018, was set. Robin

engaged Lisa as her counsel. Shortly thereafter, Joe approached Lisa at a social gathering and

brought up the case, but Lisa said “Joe I can’t talk about this because you have a lawyer.” They

spoke for approximately twenty minutes about unrelated topics. Afterwards, Lisa called Andy to

inform him that she had spoken to Joe but did not discuss the case; Andy replied, “Feel free to

talk with Joe about the case, I think everyone would be well served if we can settle.” In July,

after incurring substantial legal fees in connection with hearing preparations, Joe saw Lisa at a

restaurant and confronted her, saying “We used to be friends! This case is bankrupting me and

tearing my family apart!” Lisa replied, “Joe, I was there when your dad signed the will, and I

believe it was what he wanted. Isn’t there some way we can work this out?” Joe walked away.

Meanwhile, a new football season began and Joe resumed his practice of attending

games, drinking, and partying on Sundays. Church filed a petition asking the court to rule that

Joe forfeited his share of the estate when he violated the wishes contained in Mike’s will.

1. How is the court likely to rule on Joe’s undue influence claim?

2. Assume for this question only that the will was held to be valid. How will the

court rule on Church’s petition to divest Joe of his share in Mike’s estate?

3. What, if any, are the federal income tax consequences of Mike’s gambling during

calendar year 2018? Assume that Mike is a cash basis taxpayer, calendar year tax

filer, entitled to itemize deductions and that all of his 2018 gambling transactions

were legal and are accounted for in the fact pattern.

4. Did Lisa violate the Pennsylvania Rule of Professional Conduct governing

communications with represented persons in her interactions with Joe?

2

Page 7: JULY 2018 PENNSYLVANIA BAR EXAMINATION · JULY 2018 PENNSYLVANIA BAR EXAMINATION . Essay Questions and Examiners’ Analyses. and . Performance Test . Pennsylvania Board of Law Examiners

Question No. 1: Examiner’s Analysis

1. The court is likely to deny Joe’s claim that Robin exerted undue influence over Mike

in the drafting of his will because Joe cannot establish that Mike had a weakened

intellect.

“The resolution of a question as to the existence of undue influence is inextricably linked

to the assignment of the burden of proof.” Estate of Clark, 461 Pa. 52, 59, 334 A.2d 628, 631

(1975). After the will is admitted to probate, the burden is on the challenger to the will to

establish undue influence. Id. To establish a prima facie case for undue influence, a contestant to

a will must show the following factors: (1) confidential relationship; (2) substantial benefit; and

(3) weakened intellect. Estate of Reichel, 484 Pa. 610, 615, 400 A.2d 1268, 1270 (1979). If

these elements are proven by clear and convincing evidence, the burden of proof shifts to the

proponent of the will to show that the bequest was not the product of undue influence. Id.

A confidential relationship exists when the “parties did not deal on equal terms, but on

the one side there is an overmastering influence, or, on the other, weakness, dependence or trust,

justifiably reposed.” Estate of Clark, 461 Pa. at 63, 334 A.2d at 633 (quotation and citation

omitted).

[A] confidential relationship is not limited to any particular association of parties

but exists wherever one occupies toward another such a position of advisor or

counsellor as reasonably to inspire confidence that he will act in good faith for the

other's interest.

Brooks v. Conston, 356 Pa. 69, 76, 51 A.2d 684, 688 (1947). “[N]o clearer indication of a

confidential relationship can exist than giving another person the power of attorney over one’s

entire life savings.” Estate of Bankovich, 344 Pa. Super. 520, 524, 496 A.2d 1227, 1229 (1985)

(quotation and citation omitted).

It is likely that Joe can establish that Robin stood in a confidential relationship with Mike.

Her relationship with Mike was rooted in her capacity as his pastor, and she provided counseling

to Mike at a difficult time in his life, i.e. when he was facing sickness and death. She was also

named his agent under a power of attorney. Mike exhibited trust in and showed dependence on

Robin, and Robin was in a position of authority with respect to Mike.

It is also likely that Joe can establish that Robin received a substantial benefit from the

will. Whether the proponent of the will received a substantial benefit is determined on a case by

case basis. Estate of Fritts, 906 A.2d 601, 609 (Pa. Super. 2006). In Fritts, the court stated that

an increase from a 40% share to a 61.5 % share constituted a substantial benefit. Id. Merely

being named executor is not enough to establish a substantial benefit. Estate of Stout, 746 A.2d

645, 649 (Pa. Super. 2000). However, it is relevant whether the proponent of the will is a

relative by blood of the decedent Id. (“[W]here there is a blood relationship between the testator

and the beneficiaries of her estate, that fact alone forms a sufficient, independent basis for the

bequest.” (citation and internal quotation omitted)). Here, Robin is not a relative of Mike and

3

Page 8: JULY 2018 PENNSYLVANIA BAR EXAMINATION · JULY 2018 PENNSYLVANIA BAR EXAMINATION . Essay Questions and Examiners’ Analyses. and . Performance Test . Pennsylvania Board of Law Examiners

has been named executor. Moreover, she had no interest under his prior will, and received a one-

half interest under the will at issue. All of these factors together likely mean Robin received a

substantial benefit from Mike’s 2018 will.

Joe is unlikely to be able to establish the final element of undue influence, however.

Pennsylvania law has “not established a bright-line test by which weakened intellect can be

identified to a legal certainty,” however, “it is typically accompanied by persistent confusion,

forgetfulness and disorientation.” Estate of Fritts, 906 A.2d 601 at 607. “The weakened mental

condition which must be shown does not [need to] rise to the level of testamentary incapacity.”

Estate of Ziel, 467 Pa. 531, 542, 359 A.2d 728, 734 (1976). “Evidence of physical infirmities,

however, is not enough, alone, to establish weakened intellect.” Estate of Glover, 447 Pa. Super.

509, 517, 669 A.2d 1011, 1015 (1996).

Bodily weakness alone is not sufficient to shift the burden of proof upon the

beneficiary occupying a confidential relation to the testatrix, for one may be

physically weak and yet have a perfectly sound and strong mind.

King’s Will, 369 Pa. 523, 529-30, 87 A.2d 469, 473 (1952).

There is nothing in the facts to suggest that Mike’s mental state approached that required

to show weakened intellect. His bodily illness may have caused pain, discomfort, and stress but

there is no evidence that it affected his mental state. Nothing in the facts suggests that Mike was

confused, disoriented, or forgetful; in fact, Joe observed that Mike remained cheerful and

organized in his daily life. Because Joe cannot establish a weakened intellect, he will fail to

establish a prima facie case for undue influence. As a result, the court will likely deny Joe’s

claim.

2. The court will likely deny the petition to divest Joe of his share of Mike’s estate

because Mike’s wishes regarding Joe’s conduct as expressed in his will are

precatory.

The language in Mike’s will indicating his desire that Joe begin attending church and stop

drinking and watching football is likely to be interpreted to be precatory and not binding.

“Precatory words have been defined as words in a will expressive of desire, recommendation and

confidence.” In re Kirk’s Estate, 38 Pa. D. & C.2d 532, 534 (Pa. Com. Pl. 1965) (quotation and

citation omitted); see also The Wolters Kluwer Bouvier Law Dictionary Desk Edition (2012)

(“Language in an instrument that requests some conduct rather than commanding it.”). Mike’s

stated “wish” is not phrased as mandatory.

[T]he word "wish" . . . is generally classified as precatory. However, such a word

may be mandatory when expressive of an intention of the testator to be carried out

without the intervention of another's will and when used "in direct reference to the

estate."

In re Corbett’s Estate, 430 Pa. 54, 57, 241 A.2d 524, 525 (1968) (quoting Calder’s Estate, 343

Pa. 30, 37, 21 A.2d 907, 911 (1941).

4

Page 9: JULY 2018 PENNSYLVANIA BAR EXAMINATION · JULY 2018 PENNSYLVANIA BAR EXAMINATION . Essay Questions and Examiners’ Analyses. and . Performance Test . Pennsylvania Board of Law Examiners

While expressions of a wish will amount to a testamentary disposition if such is in

accordance with the intention of the testator, yet, where precatory words follow an

absolute disposition of the property, it has been established as a rule of property in

Pennsylvania that such precatory words will not operate to diminish the absolute

estate previously given.

Chew v. Chew, 266 Pa. 526, 528, 109 A. 799, 800 (1920); see also Sommerville Estate, 417 Pa.

600, 605, 209 A.2d 299, 302 (1965) (“An absolute estate will not be cut down by subsequent

language unless the testator has expressed a clear and unambiguous intention to do so.”).

Mike’s gift to Joe is absolute – the will specifically states that Joe may do with the

bequest “as he chooses.” The will also provides that if Joe predeceases Mike, his share goes to

New Pennsylvania Church, but does not require the same result if Joe fails to follow his

expressed wishes. Moreover, Mike’s expression of desire for his son’s future conduct does not

directly mention his estate. Therefore, the general interpretation of “wishes” will control, and

the court will likely determine that the words regarding Joe’s behavior are precatory.

Accordingly, the court will dismiss Church’s petition.

3. Mike has taxable income of $30,000 from gambling in 2018.

Gross income generally includes “all income from whatever source derived.” 26 U.S.C.

§ 61. This includes income from gambling winnings. See e.g. Carmack v. Commissioner, 183

F.2d 1, 2 (5th Cir. 1950) (stating poker winnings to be included in taxable income); Bodine v. Commissioner, T.C. Memo 1984-143 (holding gambling winnings included in income even

where tax was withheld). “Losses from wagering transactions shall be allowed only to the extent

of the gains from such transactions.” 26 U.S.C.S. § 165(d); 26 C.F.R. § 1.165-10.

Mike had a net $40,000 in gambling winnings prior to the football championship game,

and he lost $10,000 on that game. These transactions represent all of Mike’s gambling activity

for the year. Under section 61 Mike will have to report his net $40,000 in gambling winnings on

his 2018 taxes.

Then, pursuant to section 165(d), Mike will be entitled to deduct wagering losses from

his gambling gains. The question indicates that Mike may itemize his deductions for the

calendar year. After deducting his championship game loss of $10,000 from his gambling gains

of $40,000, Mike is left with $30,000 of taxable income from gambling transactions for calendar

year 2018. The federal tax consequences of Mike’s gambling activity are income of $40,000,

and a deduction of $10,000, or net income of $30,000.

4. Lisa did not violate the Pennsylvania Rule of Professional Conduct in her

communications with Joe.

The rules of Professional Conduct regulate communication between lawyers and

represented parties. “In representing a client, a lawyer shall not communicate about the subject

5

Page 10: JULY 2018 PENNSYLVANIA BAR EXAMINATION · JULY 2018 PENNSYLVANIA BAR EXAMINATION . Essay Questions and Examiners’ Analyses. and . Performance Test . Pennsylvania Board of Law Examiners

of the representation with a person the lawyer knows to be represented by another lawyer in the

matter, unless the lawyer has the consent of the other lawyer or is authorized to do so by law or a

court order.” Pa. R.P.C. 4.2. “This Rule does not prohibit communication with a represented

person, or an employee or agent of such a person, concerning matters outside the representation.”

Pa. R.P.C. 4.2 cmt. 4. “The Rule applies even though the represented person initiates or consents

to the communication.” Pa. R.P.C. 4.2 cmt. 3.

The first time Lisa spoke with Joe after becoming involved in the case, she was aware

that Joe was represented by Andy, as she demonstrated by telling Joe she could not discuss the

case. As such, she did not communicate with him about the subject of the representation. The

first communication was permissible under Rule 4.2.

By the time of the second communication, during which Lisa did communicate with Joe

about the subject of the representation, Andy had authorized Lisa to communicate with Joe about

the case. Therefore, Lisa did not violate any rule of professional conduct in either of her

communications with Joe.

6

Page 11: JULY 2018 PENNSYLVANIA BAR EXAMINATION · JULY 2018 PENNSYLVANIA BAR EXAMINATION . Essay Questions and Examiners’ Analyses. and . Performance Test . Pennsylvania Board of Law Examiners

Question No. 1: Grading Guidelines

1. Undue influence.

Comments: Applicants should identify and apply the elements required to state a prima facie

case for undue influence.

7 points.

2. Precatory words.

Comments: Applicants should recognize the rule that precatory words are not binding on the

beneficiaries of a will.

5 points.

3. Taxation of gambling gains and losses.

Comments: Candidates should recognize that gambling gains are included in gross income, and

that gambling losses can be deducted.

3 points.

4. Communication with a represented person.

Comments: Candidates should identify the rule prohibiting communication with a represented

person concerning the subject matter of the representation and the exception where counsel for

such person gives consent to the communication.

5 points.

7

Page 12: JULY 2018 PENNSYLVANIA BAR EXAMINATION · JULY 2018 PENNSYLVANIA BAR EXAMINATION . Essay Questions and Examiners’ Analyses. and . Performance Test . Pennsylvania Board of Law Examiners

Question No. 2

In January 2018, Diane decided to open a commercial photography studio from which

she could take photographs for sale and could produce custom posters for events. Subsequently,

Diane decided to form a corporation, Photos, Inc. (“Photos”), to be the operator of the business.

In January, Diane negotiated a lease for a commercial space in Pennsylvania with Realty,

a Pennsylvania company. Diane told Realty she was acting on behalf of Photos. The fully

executed written lease named Photos as tenant and was signed by Diane as president of Photos.

In April, Diane, without consulting an attorney, incorporated Photos as a Pennsylvania

corporation. She is Photos’ sole shareholder. Photos has no bylaws, minutes, issued stock

certificates, letterhead or business cards. Diane did not contribute any assets to Photos other than

$200 in cash. She has been using a camera that she owns personally. Photos has a checking

account from which Diane routinely pays both Photos’ expenses and her personal expenses.

On July 1, Photos was awarded a contract to produce customized posters for each senior

athlete at a local Pennsylvania high school to be presented to the athletes at an assembly on the

first day of school in late August. The contract required delivery of the posters by August 20 and

expressly provided, that if all posters are not delivered by August 20, the contract would be void

and the school would have no obligation to pay Photos for any work done.

Diane concluded that Photos would need a new laser printer to produce the posters on a

timely basis. On July 2, she contacted Printers, Inc. (“Printers”), a manufacturer and seller of

printers and discussed Photos’ need for a printer for its commercial operation. Diane, as agent of

Photos, purchased a new laser printer. The written sales agreement, properly executed and

signed by Printers and Photos, provided, in part:

The buyer’s exclusive remedy against the seller for failure of or defect in the

printer shall be for the repair or replacement of defective parts. No other remedy

8

Page 13: JULY 2018 PENNSYLVANIA BAR EXAMINATION · JULY 2018 PENNSYLVANIA BAR EXAMINATION . Essay Questions and Examiners’ Analyses. and . Performance Test . Pennsylvania Board of Law Examiners

(including, without limitation, incidental or consequential damages for lost

profits, lost sales, or any other incidental or consequential loss) shall be available

to buyer. The buyer has signed this provision to evidence its understanding and

agreement that the remedy provided for is the sole and exclusive remedy available

to the buyer in the event of failure of the printer. Diane Buyer’s signature

The printer was delivered July 10, set up, tested and accepted by Photos. The printer was

initially working fine but, three days ago, stopped working. Diane promptly called Printers, and

Printers immediately sent out a technician. It was determined that the circuit board in the printer

was defective and needed to be replaced. Diane demanded a new printer or her money back

because the printer provided had failed in breach of the warranty of merchantability. Printers

refused and advised it would promptly install a new circuit board in the printer. Diane,

concerned about the delay the repair of the printer would cause, contacted the school and advised

the posters would not be delivered until August 30. The school indicated that if all of the posters

were not delivered by August 20 the contract would be void. Printers installed a new circuit

board yesterday and the printer is again working fine.

Photos is really counting on the school contract as it has fallen behind in paying rent,

utilities, and other bills. Due to these events, Diane is contemplating going out of business.

1. If Photos ceases operations, what two theories could be advanced by Realty to

support a claim against Diane personally in an action to collect past due rent

based, first, upon Diane’s pre-incorporation activities and, second, her status as

shareholder of Photos?

2. Under the Pennsylvania Uniform Commercial Code (the “Code”), when Diane

demanded a replacement printer or her money back was Printers within its rights

under the contract to refuse her request and proceed with steps to repair and

replace the defective circuit board?

3. Assume for this question that Photos has not produced and delivered all of the

posters and the school validly declares the contract void per the contract terms.

Under the Code, can Photos successfully recover its lost profits from Printers if it

asserts and proves that its failure to meet the deadline was due to the failure of the

laser printer?

9

Page 14: JULY 2018 PENNSYLVANIA BAR EXAMINATION · JULY 2018 PENNSYLVANIA BAR EXAMINATION . Essay Questions and Examiners’ Analyses. and . Performance Test . Pennsylvania Board of Law Examiners

Question No. 2: Examiner’s Analysis

1. Realty could argue that Diane has liability under the lease as a promoter and could

also argue the equitable theory of piercing the corporate veil of Photos in an effort

to find Diane personally liable under the lease.

Realty could argue that when the lease was signed Diane was acting as a promoter of

Photos. A “promoter” is a person who purports to act on behalf of a proposed corporation that

has not yet been incorporated. Generally, a promoter is someone who takes an active part in

creating and organizing the corporation. An officer who executes or enters into a pre-

incorporation contract may have the legal status of a promoter. Fletcher Cyc. Corp. § 189 (Perm.

Ed. 1999). In the absence of an express or implied agreement to the contrary, a promoter is liable on

a pre-incorporation contract even though the contract purports to be on behalf of the corporation

to be formed. Id. at § 215. “Even though they purport to act on behalf of the proposed

corporation and not for themselves, promoters may be held personally liable on contracts made

by them prior to the actual formation of the corporation. In the absence of a novation or an

agreement by the other party to a release of liability, the promoter will remain liable after the

corporation is formed.” John W. McLamb & Wendy C. Shiba, Pennsylvania Corporate Law &Practice § 2.2 (1993).

With respect to the lease agreement with Realty, Diane was clearly acting as a promoter.

She negotiated the lease with Realty purporting to enter into the lease contract on behalf of a

corporation (Photos) that had not yet been formed. There is no indication that Realty agreed to

look only to Photos to enforce its rights under the lease or to release Diane if and when Photos

was formed. Realty should be successful in asserting a claim directly against Diane if it so

desires based upon her status as a promoter. See, RKO-Stanley Warner Theaters, Inc. v. Graziano, 467 Pa. 220, 355 A.2d 830 (1976).

Realty could also argue that it should be permitted to pierce the corporate veil of Photos

and collect the past due rent directly from Diane as Photos’ sole shareholder.

Generally, a corporation is an entity separate and apart from its shareholders even if all of the

stock of the corporation is owned by one individual or entity. Barium Steel Corp. v. Wiley, 379 Pa.

38, 47, 108 A.2d 336, 341 (1954). Courts will uphold the separate corporate existence in most cases

and leave the shareholder free of liability for the corporation’s debts. John W. McLamb, Jr. & Wendy C. Shiba, Pennsylvania Corporate Law & Practice, § 5.9 (1993). There are, however, certain

instances when the corporate veil will be pierced and the general rule of shareholder protection will

not apply.

There are no precise rules or tests to be applied to determine if the corporate veil should be

pierced. Barium Steel Corp., 379 Pa. at 47, 108 A.2d at 341. Piercing a corporate veil is equitable in

nature and is driven by the circumstances of each case. “There are, however, two [overarching]

elements required by most jurisdictions. First, there must be such unity of interest and ownership

that the separate personalities of the corporation and the individual no longer exist[. S]econd, the

10

Page 15: JULY 2018 PENNSYLVANIA BAR EXAMINATION · JULY 2018 PENNSYLVANIA BAR EXAMINATION . Essay Questions and Examiners’ Analyses. and . Performance Test . Pennsylvania Board of Law Examiners

circumstances must indicate that adherence to the fiction of separate corporate existence would

sanction a fraud or promote injustice.” Fletcher Cyc. Corp., supra § 41.30.

Although the analysis is somewhat subjective, there are general elements that have been

identified by courts that support a finding that the corporate veil should be pierced. These factors

include, whether the corporation is inadequately capitalized, failed to observe corporate formalities,

failed to issue stock, failed to pay dividends, operates without profit, commingles corporate assets,

with personal assets of its shareholder(s) and is devoid of corporate records. Id., see also, Lumax Industries v. Aultman, 543 Pa. 38, 669 A.2d 893 (1995). The Pennsylvania Supreme Court stated,

“the corporate veil is properly pierced whenever one in control of a corporation uses that control or

corporate assets to further one’s own personal interests.” College Watercolor Group, Inc. v. Newbauer, 468 Pa. 103, 117, 360 A.2d 200, 207 (1976). Thus, if the corporation exists merely in

name only and is used as the alter ego of the shareholder a court may pierce the corporate veil and

impose liability directly on the shareholder.

In this case, Photos has minimal assets. Diane has failed to observe any corporate

formalities. Photos has no bylaws, minutes, or issued stock certificates. It has no letterhead or

business cards. Even though it has a corporate checking account, Diane routinely uses funds in that

account to pay her personal expenses which acts to the detriment of Photos’ creditors. Based on

these factors, Realty has a basis to seek to have the corporate veil pierced, should the promoter

liability action prove unsuccessful, and may be successful in doing so.

2. Printers most likely had the contractual right to refuse to replace the failed printer

with a new one or to refund Photos’ money and to proceed with repair and

replacement of the defective circuit board.

When the printer failed, Diane demanded a new printer or her money back based upon a

belief that the failure of the printer constituted a breach of the implied warranty of

merchantability. She may be correct in her assertion of breach of warranty. The facts are silent

as to existence of any disclaimer of this implied warranty. The issue presented here, however, is

not one of the applicability of an implied warranty, which is most likely implicated by the facts;

but, instead is one of what remedies are available to the buyer as a result of a breach. If this

warranty was breached, normally, provided any and all procedural steps are properly taken,

rejection and return of the good and replacement or refund of the purchase price are remedies

that could be available under the Code. See 13 Pa. C.S.A. §§ 2601, 2608. Here, Printers’

position is that the contract defines the exclusive remedy available as the repair and replacement

of the defective part.

Section 2719 of the Code addresses this situation. This section provides:

(a) General rule .-- Subject to the provisions of subsections (b) and (c) and of

section 2718 (relating to liquidation or limitation of damages; deposits):

(1) The agreement may provide for remedies in addition to or in substitution for

those provided in this division and may limit or alter the measure of damages

recoverable under this division, as by limiting the remedies of the buyer to return

11

Page 16: JULY 2018 PENNSYLVANIA BAR EXAMINATION · JULY 2018 PENNSYLVANIA BAR EXAMINATION . Essay Questions and Examiners’ Analyses. and . Performance Test . Pennsylvania Board of Law Examiners

of the goods and repayment of the price or to repair and replacement of

nonconforming goods or parts.

(2) Resort to a remedy as provided is optional unless the remedy is expressly

agreed to be exclusive, in which case it is the sole remedy.

(b) Exclusive remedy failing in purpose. -- Where circumstances cause an

exclusive or limited remedy to fail of its essential purpose, remedy may be had as

provided in this title.

(c) Limitation of consequential damages. -- Consequential damages may be

limited or excluded unless the limitation or exclusion is unconscionable.

Limitation of consequential damages for injury to the person in the case of

consumer goods is prima facie unconscionable but limitation of damages where

the loss is commercial is not.

13 Pa. C.S.A. §2719. Generally, limitations on remedies provided in a sales contract are optional unless the

agreement expressly provides that the remedy or remedies are to be exclusive. 13 Pa. C.S.A.

§2719(a)(2). Comment 2 to section 2719 provides,

Subsection (1)(b) creates a presumption that clauses prescribing remedies are

cumulative rather than exclusive. If the parties intend the term to describe the sole

remedy under the contract, this must be clearly expressed.

In this case, the contract clearly provides “the buyer’s exclusive remedy . . . shall be . . . repair

and replacement of defective parts.” This fact was made clear to the buyer as the contract

required the buyer to sign the limitation of remedy clause of the contract. On its face, it would

appear that this is the only remedy that the buyer can request and demand. The analysis,

however, does not end there. The remedy must be further analyzed under Subsection (b); i.e.,

does the remedy “fail of its essential purpose” and it must be determined if the remedy and the

exclusion of all other remedies is unconscionable.

Subsection (b) of 2719 provides, “Where circumstances cause an exclusive . . . remedy to

fail of its essential purpose, remedy may be had as provided in this title.” In other words, if the

remedy fails of its essential purpose then a court may determine that it is not the exclusive

remedy available to the buyer and that the buyer may resort to other remedies available under the

Code. Comment 1 to section 2719 provides, in part:

[I]t is the very essence of a sales contract that at least minimum adequate

remedies be available. If the parties intend to conclude a contract for sale within

this Article they must accept the legal consequence that there be at least a fair

quantum of remedy for breach of the obligations or duties outlined in the contract.

A remedy fails of its essential purpose if it fails to provide a remedy that gives the buyer the

benefit of its bargain and if it fails to do exactly what it is intended to do; i.e., repair the printer.

12

Page 17: JULY 2018 PENNSYLVANIA BAR EXAMINATION · JULY 2018 PENNSYLVANIA BAR EXAMINATION . Essay Questions and Examiners’ Analyses. and . Performance Test . Pennsylvania Board of Law Examiners

13 Pa. C.S.A. §2719, cmt. 1. Normally, the buyer must give the seller a reasonable opportunity

to effectuate the remedy and the seller must, in fact, effectuate the remedy curing the problem.

See White and Summers, Uniform Commercial Code, §12-10 a. (4th Ed. 1995). In other words

the buyer must give the seller notice of the problem and allow the seller an opportunity to

remedy the situation.

In this case, when the printer stopped working, the buyer notified the seller who

immediately sent out a technician who identified the defective circuit board as the problem.

Within days, the circuit board was replaced and the printer was again working as it should.

Although the determination of whether or not a remedy “fails its essential purpose” is looked at

on a case-by-case basis a strong argument can be made by Printers that the remedy was adequate

and did not fail in its essential purpose. The remedy did what it was supposed to do. It resulted

in the prompt repair of the printer. See New York State Elec. & Gas Corp. v. Westinghouse Elec. Corp., 387 Pa. Super. 537, 557, 564 A.2d 919, 929 (1989). Should a court find otherwise, then

the exclusive nature of the remedy would fail and the buyer could resort to other remedies. See, White and Summers, Uniform Commercial Code, § 12-10 (4th Ed. 1995).

Comment 1 to Section 2719 further provides “any clause purporting to modify or limit

the remedial provisions of this Article in an unconscionable manner is subject to deletion and in

that event the remedies made available by this Article are applicable as if the stricken clause had

never existed.” Section 2302, which addresses unconscionability, provides:

(a) Finding and authority of court.--If the court as a matter of law finds the

contract or any clause of the contract to have been unconscionable at the time

it was made, the court may:

(1) refuse to enforce the contract;

(2) enforce the remainder of the contract without the unconscionable clause; or

(3) so limit the application of any unconscionable clause as to avoid any

unconscionable result.

(b) Evidence by parties.--When it is claimed or appears to the court that the

contract or any clause thereof may be unconscionable the parties shall be afforded

a reasonable opportunity to present evidence as to its commercial setting, purpose

and effect to aid the court in making the determination.

Comment 1 to Section 2302 explains, “The basic test is whether, in the light of the general

commercial background and the commercial needs of the particular trade or case, the clauses

involved are so one-sided as to be unconscionable under the circumstances existing at the time of

the making of the contract.” Unconscionability is a somewhat subjective standard without clear

lines that delineate when something is or not unconscionable. A court would hear testimony and

would have to determine if the limitation of the remedy, as set forth in the contract, is or is not

unconscionable. In a commercial setting a court will rarely find a contract clause

unconscionable. “The principal underlying a finding of unconscionability is to prevent

oppression and unfair surprise . . . .” Jim Dan, Inc. v. O. M. Scott & Sons Company, 785 F.

13

Page 18: JULY 2018 PENNSYLVANIA BAR EXAMINATION · JULY 2018 PENNSYLVANIA BAR EXAMINATION . Essay Questions and Examiners’ Analyses. and . Performance Test . Pennsylvania Board of Law Examiners

Supp. 1196, 1200 (W.D. Pa. 1992). In this case, the remedy was clearly called out in the

contract and was acknowledged and accepted by the buyer. When a problem occurred, the

remedy proved to be effective, as the printer was repaired by replacement of the defective part

and was back up and running within a week. Although reasonable minds could disagree, a good

argument could be made that the limitation of remedy is not unconscionable and should be

upheld by a court.

3. A sales contract may limit the availability of consequential damages unless the

limitation or exclusion is unconscionable and, therefore, it is unlikely that Photos

will recover for its lost profits.

As set forth above, 2719(c) provides, “Consequential damages may be limited or

excluded unless the limitation or exclusion is unconscionable. Limitation of consequential

damages for injury to the person in the case of consumer goods is prima facie unconscionable but

limitation of damages where the loss is commercial is not.” 13 Pa.C.S.A. § 2719(c). In the

instant case we are not dealing with a sale of consumer goods to a consumer and the claim would

not be one asserted for injury to a person. Thus, the second sentence of 2719(c) would not be

applicable. This was a sale to a commercial buyer for use in a commercial enterprise.

The question asked is whether Photos could recover lost profits. Section 2715 of the

Code defines “consequential damages” to include “any loss resulting from general or particular

requirements and needs of which the seller at the time of contracting had reason to know and

which could not reasonably be prevented by cover or otherwise.” 13 Pa.C.S.A. § 2715(b).

Printers knew that the printer was being purchased by a commercial enterprise to print posters.

Thus, loss of profits due to the failure or inability to print posters was or should have been

known to Printers.

The question, then, is the effect of the exclusion of consequential damages in the

contract. Generally, commercial parties are and should be free to contract as they wish. See, White and Summers, Uniform Commercial Code § 12-10 c. (4th Ed. 1995). This general rule, of

course, is tempered by the general application of the unconscionability provisions of the Code.

See Section 2302 discussion above. In this case, the parties clearly agreed to the limitation. This

case does not deal with a consumer transaction. Given the precept that parties, particularly

commercial parties, should be free to contract as they wish, a strong argument can be made that

the exclusion of consequential damages should apply and that Photos would not be able to

recover lost profits. A further argument could be made by Printers that the minimal down time

should not have prevented Photos from completing its project on time. Thus the lost profits

being claimed were a result of Photos own doing and not as a result of the limited down time of

the printer. See, White and Summers, Uniform Commercial Code § 12-10 c. (4th Ed. 1995).

14

Page 19: JULY 2018 PENNSYLVANIA BAR EXAMINATION · JULY 2018 PENNSYLVANIA BAR EXAMINATION . Essay Questions and Examiners’ Analyses. and . Performance Test . Pennsylvania Board of Law Examiners

Question No. 2: Grading Guidelines

1. Business Organizations – Promoter liability and piercing the corporate veil.

Comments: The candidates should analyze Diane’s potential liability under the lease as a

promoter and a shareholder (applying piercing the corporate veil principles).

7 points

2. Sales – Limitation of remedy to repair or replacement of defective parts

Comments: The candidates should discuss the ability of a seller to limit remedies to an exclusive

remedy or remedies and the limitations on a seller’s ability to do so under the Uniform

Commercial Code; i.e., does the remedy fail of its essential purpose or is the remedy

unconscionable.

7 points

3. Sales – Limitation of consequential damages is a commercial contract

Comments: The candidates should discuss the ability of a seller in a commercial contract to limit

consequential damages, and in particular, lost profits, and the limitation on the ability to do so,

i.e., unconscionability.

6 points

15

Page 20: JULY 2018 PENNSYLVANIA BAR EXAMINATION · JULY 2018 PENNSYLVANIA BAR EXAMINATION . Essay Questions and Examiners’ Analyses. and . Performance Test . Pennsylvania Board of Law Examiners

Question No. 3

Ted and Kayla were married in 1988 and have owned and continuously lived in a home

located in C County, Pa. Their one daughter, Mackenzie, turned 17 years of age on June 15,

2017. Unbeknownst to Ted, Kayla has allowed Mackenzie to have several beer parties at their

home while Ted was away on business. These parties have been attended by teenagers who have

consumed alcohol to the point of intoxication and Kayla, who has been present and provided the

beer for all of the parties, has allowed the teenagers to drive home after the parties.

On Saturday afternoon, June 17, 2017, while Ted was away on business, Kayla purchased

two kegs of beer from a beer distributor in C County by using her credit card. Kayla put the

purchase receipt in her pocket. The receipt had the following information on it: distributor name,

date and time of purchase, type and amount of beer purchased, the cost of the purchase, and

Kayla’s credit card number. Kayla proceeded home and had several of Mackenzie’s teenage

friends carry the kegs down to her basement where they iced them down for that evening’s party.

That evening, about 40 of Mackenzie’s teenage friends came over and consumed beer for

several hours. Some of the friends went upstairs and talked with Kayla while drinking their beer.

At midnight, John, Frank, Heidi, and Chrissy all left in the same car with Frank driving. Frank,

who spoke with Kayla before leaving and was visibly intoxicated, drove about two miles before

he failed to negotiate a curve in the road and hit a tree. Frank was killed instantly and the

remaining passengers were seriously injured but remained conscious. When the police arrived at

the accident scene, Heidi told them that they were at a party at Mackenzie’s house and that her

mother, Kayla, had bought the beer for the party.

After determining Kayla’s address, the police immediately went to her home. Kayla told

the police that she does not permit underage drinking at her home. Based upon the above

16

Page 21: JULY 2018 PENNSYLVANIA BAR EXAMINATION · JULY 2018 PENNSYLVANIA BAR EXAMINATION . Essay Questions and Examiners’ Analyses. and . Performance Test . Pennsylvania Board of Law Examiners

information given by Heidi, the police lawfully placed Kayla under arrest and charged her with

furnishing alcohol to minors and for involuntary manslaughter for Frank’s death. Kayla was

taken to the police station and, as part of the established booking process, her clothing was

searched resulting in the recovery of the receipt for the beer purchase on June 17, 2017. The

morning after the accident, Ted learned that Kayla had hosted the party and had done so in the

past. Ted went to the police station, where Kayla was being held, and he told Kayla he wanted a

divorce and had moved out. Kayla responded she would never agree to a divorce. Subsequent

investigation revealed that Frank was legally intoxicated at the time of the accident based on the

beer he consumed at Kayla’s home and that his intoxication caused the accident.

1. Do the facts support a charge of involuntary manslaughter against Kayla for

Frank’s death?

Assume Kayla proceeds to trial on the charges filed. At trial, the Commonwealth, after

having timely informed the defense before trial of its intention to do so, attempts to admit into

evidence that Kayla hosted several underage drinking parties prior to June 17, 2017, to establish

that she knew there was underage drinking taking place at her home. The defense objects to this

evidence on the basis that these prior incidents should not be admitted to prove that Kayla was

responsible for Frank’s death.

2. Aside from any habit argument, what argument(s) should the Commonwealth

make to support the admission of this evidence?

3. If Kayla’s attorney challenges the search and seizure that resulted in the police

recovering the beer purchase receipt on the basis that the police lacked a warrant,

how should the Commonwealth respond and how would the court likely rule?

4. Assume that Kayla is found guilty of both charges on July 7, 2018, and is

immediately sentenced to incarceration for a minimum sentence of 3 years.

Based on all the facts presented, what grounds for divorce does Ted have as of

July 7, 2018, to end his marriage to Kayla?

17

Page 22: JULY 2018 PENNSYLVANIA BAR EXAMINATION · JULY 2018 PENNSYLVANIA BAR EXAMINATION . Essay Questions and Examiners’ Analyses. and . Performance Test . Pennsylvania Board of Law Examiners

Question No. 3: Examiner’s Analysis

1. The facts do support a charge of involuntary manslaughter against Kayla for Frank’s

death due to Kayla’s reckless conduct.

“A person is guilty of involuntary manslaughter when as a direct result of the doing of an

unlawful act in a reckless or grossly negligent manner, or the doing of a lawful act in a reckless

or grossly negligent manner, he causes the death of another person.” 18 Pa.C.S.A. § 2504 (a).

“[I]nvoluntary manslaughter requires (1) a mental state of either recklessness or gross negligence

and (2) a causal nexus between the conduct of the accused and the death of the victim.”

Commonwealth v. McCloskey 835 A.2d 801, 806 (Pa. Super. 2003). The McCloskey court went

on to explain the following:

[R]ecklessness is statutorily defined as “consciously disregarding a substantial

and unjustifiable risk that the material element exists or will result from his

conduct. The risk must be of such a nature and degree that, considering the nature

and intent of the actor’s conduct and the circumstances known to him, its

disregard involves a gross deviation from the standard of conduct that a

reasonable person would observe in the actor’s situation.”

McCloskey, 835 A.2d at 806 quoting 18 Pa.C.S.A. § 302 (b)(3).

The Pennsylvania Supreme Court has construed the terms “reckless” and “grossly negligent”

as defining the equivalent state of mind for purposes of the involuntary manslaughter provision.

Commonwealth v. Huggins, 836 A.2d 862, 868 (2002) citing Commonwealth v. Comer, 552 Pa.

527, 716 A.2d 593, 597 (Pa. 1998) (definition of “recklessly” set forth in Section 302

“encompasses” gross negligence set forth in Section 2504) (citing Commonwealth v. Lobiondo,

501 Pa. 599, 462 A.2d 662 (Pa. 1983)).

In McCloskey, the court was faced with facts similar to the current case, in that

McCloskey (the mother) permitted her underage daughter to host an underage drinking party and

assisted in the planning of and preparation for the party. The mother was home for the entire

party and witnessed several teenagers drinking beer. The court concluded:

[A] parent who knows alcohol is being served to minors in her home is acting

recklessly when she allows the conduct to continue. That knowledge not only

constitutes a “gross deviation from the standard of conduct that a reasonable

person would observe in “her” situation,” 18 Pa. C.S.A. § 302 (b)(3), it also

constitutes a clear violation of the law. 18 Pa. C.S.A. § 6310.1(a).

McCloskey, 835 A.2d at 807.

The McCloskey court, quoting Commonwealth v. Nicotra, 625 A.2d 1259, 1264 (Pa.

Super. 1993), pointed out that the courts of this Commonwealth have held that “‘so long as the

defendant’s conduct started the chain of causation which led to the victim’s death, criminal

18

Page 23: JULY 2018 PENNSYLVANIA BAR EXAMINATION · JULY 2018 PENNSYLVANIA BAR EXAMINATION . Essay Questions and Examiners’ Analyses. and . Performance Test . Pennsylvania Board of Law Examiners

responsibility for the crime of homicide may properly be found.’” McCloskey, 835 A.2d at 808.

The court went on to conclude that when the mother furnished alcohol to the minor victim, she

started the chain of causation that led to the death of all of the victims. McCloskey, 835 A.2d at

808.

As applied here, Kayla purchased two kegs of beer for a party which she knew was going

to be attended by multiple teenagers. Kayla had several of the teenagers carry the beer down to

her basement and ice them down. Kayla had full knowledge that the teenagers were drinking as

the facts indicate that some of them were drinking in her presence. She also observed that Frank

was visibly intoxicated before he left her premises. This conduct would likely support the

mental state of recklessness or gross negligence for involuntary manslaughter as Kayla

consciously disregarded the substantial and unjustifiable risk that her conduct might lead to the

injury or death of another when she allowed Frank to drink and then drive in a visibly intoxicated

state. This would constitute a gross deviation from the standard of conduct that a reasonable

person should have observed in her situation.

The facts also indicate that the investigation into the accident revealed that Frank was

legally intoxicated at the time of the accident having consumed all of his alcohol at Kayla’s

home and that his intoxication caused the accident. Because Kayla permitted the minors,

including Frank, to consume alcohol and allowed Frank to drive, she started the chain of

causation which led to Frank’s death. Therefore, it is likely that there was a sufficient causal

nexus between Kayla’s conduct and Frank’s death to support a charge of involuntary

manslaughter.

In summary, it appears that the charge of involuntary manslaughter would be supported

by the facts presented.

2. The Commonwealth should argue that Kayla’s hosting of prior underage drinking

parties would be admissible under Pa.R.E. 404(b), otherwise known as prior bad act

evidence, and that the probative value of this evidence outweighs its prejudicial

effect.

The McCloskey court has stated the following:

The Pennsylvania Rules of Evidence govern the admission of prior bad acts

evidence. The relevant rule provides that “evidence of other crimes, wrongs or

acts is not admissible to prove the character of a person in order to show action in

conformity therewith” but may be admitted for “other purposes, such as proof of

motive, opportunity, intent, preparation, plan, knowledge, identity or absence of

mistake or accident.” Pa. R.E. 404 (b)(1) and (2). Admission is proper only if the

probative value of the evidence outweighs its potential for prejudice. Pa. R.E.

404 (b)(3).

McCloskey, 835 A.2d at 809-10, see also, Commonwealth v. Paddy, 800 A.2d 294 (Pa. 2002),

(holding that evidence of a defendant’s prior criminal activity is inadmissible to demonstrate bad

character or criminal propensity but may be admissible for various limited purposes provided

19

Page 24: JULY 2018 PENNSYLVANIA BAR EXAMINATION · JULY 2018 PENNSYLVANIA BAR EXAMINATION . Essay Questions and Examiners’ Analyses. and . Performance Test . Pennsylvania Board of Law Examiners

that its probative value outweighs the prejudicial effect likely to result from its admission and an

appropriate limiting instruction is given).

In a criminal case the prosecutor must provide reasonable notice in advance of trial, or

during trial if the court excuses pre-trial notice on good cause shown, of the general nature of any

such evidence the prosecutor intends to introduce at trial. Pa. R.E. 404(b)(3).

In McCloskey, the court was faced with the situation where teens had regularly met at the

home of a mother who hosted underage drinking parties and the mother was aware that the teens

were drinking alcohol. The court found that this evidence was offered to show the mother’s

knowledge that her residence was routinely used by minors as a place to drink alcohol. The

McCloskey court found that “the evidence . . . was probative of her awareness or knowledge of

the unlawful behavior and, as a result, was relevant to establish that she acted recklessly.” Id. at

810. It was also determined that the “trial court recognized the prejudicial nature of this

evidence and assessed its probative value against its prejudicial effect.” The Superior Court held

that the evidence was properly admitted and that the trial court did not abuse its discretion. Id.

As applied here, the facts indicate that the Commonwealth timely informed the defense

before trial of its intention to use the proposed bad act evidence. The facts further indicate that

Kayla hosted several underage drinking parties before the June 17, 2017, party that resulted in

Frank’s death. This evidence would be probative of her awareness or knowledge that teens were

regularly drinking alcohol at her home, which would be relevant to establish that she acted

recklessly under the circumstances. The Commonwealth should be prepared to argue that any

prejudice which might result from the admission of this evidence would be outweighed by the

probative value of the evidence to establish the element of recklessness for involuntary

manslaughter. Although this would ultimately be something that would have to be weighed by

the court, the Commonwealth would have persuasive arguments to admit this prior bad act

evidence on the facts presented. If the court agrees with the Commonwealth’s argument an

appropriate limiting instruction would need to be given to the jury.

3. The Commonwealth should respond that the receipt was discovered as a result of a

properly executed inventory search that is a recognized exception to the warrant

requirement; and the court should conclude that this was a legal search and that the

receipt is admissible.

“Under both the Fourth Amendment of the United States Constitution and Article I,

Section 8 of the Pennsylvania Constitution, a search conducted . . . without a warrant is deemed

to be per se unreasonable.” Commonwealth v. Cleckley, 738 A.2d 427, 429 (Pa. 1999) citing

Commonwealth v. Williams, 692 A.2d 1031 (Pa. 1997). The Pennsylvania Supreme Court stated

the following:

[I]nventory searches are a well-defined exception to the warrant requirement of the

Fourth Amendment and are a recognized part of our law: . . . “it is reasonable for police

to search the personal effects of a person under lawful arrest as part of the routine

administrative procedure at a police station house incident to booking and jailing the

suspect. The justification for such searches does not rest on probable cause, and hence

20

Page 25: JULY 2018 PENNSYLVANIA BAR EXAMINATION · JULY 2018 PENNSYLVANIA BAR EXAMINATION . Essay Questions and Examiners’ Analyses. and . Performance Test . Pennsylvania Board of Law Examiners

the absence of a warrant is immaterial to the reasonableness of the search. Indeed, we

have previously established that the inventory search constitutes a well-defined exception

to the warrant requirement.”

Commonwealth v. Nace, 571 A.2d 1389, 1391 (Pa. 1990) quoting South Dakota v. Opperman,

428 U.S. 364(1976).

According to the Nace court there are four goals that underlie such searches:

First, they protect the defendant’s property while he is in custody; second, police

are protected against theft claims when defendants are given their property upon

release; third, they serve to protect the police from physical harm due to hidden

weapons; and fourth, when necessary they ascertain or verify the identity of the

defendant.

Nace, 571 A.2d at 1391.

As applied here, Kayla was lawfully placed under arrest and was taken to the police

station. The facts indicate that as part of the established booking process her clothing was

searched resulting in the recovery of the receipt for the beer she purchased on June 17, 2017. A

search warrant would normally be required for the search of Kayla’s person. However, because

this search was part of a routine administrative procedure at the police station incident to her

booking, it would likely fall under the inventory search exception to the warrant requirement.

Because this receipt was properly obtained pursuant to the inventory search exception to the

search warrant requirement, it is likely that the court would rule that the receipt is admissible.

4. Ted likely has grounds for a fault-based divorce since Kayla has been sentenced to

imprisonment for a term of more than two years and Ted also has grounds for a no-

fault divorce based on irretrievable breakdown of the marriage as the couple have

been separated for more than one year.

The Domestic Relations Code, 23 Pa. C.S.A. § 3301(a)(5), provides that a “court may

grant a divorce to the innocent and injured spouse whenever it is judged that the other spouse has

. . . [b]een sentenced to imprisonment for a term of two or more years upon conviction of having

committed a crime.” As applied here, Ted is the innocent spouse. He was innocent of assisting

Kayla in her activities of hosting teenage drinking parties. See Murphy v. Murphy, 205 A.2d 647

(Pa. Super. 1964). Also, the facts indicate that Kayla has been convicted of the crimes charged

and has received a minimum sentence of three years of incarceration. Since this exceeds the

minimum two year imprisonment requirement under section 3301(a)(5) required to secure a

fault-based divorce, it is likely Ted would be able to secure a divorce from Kayla.

Alternatively, Ted could pursue a divorce under 23 Pa. C.S.A. § 3301(d)(1) that provides

that a “court may grant a divorce where a complaint has been filed alleging that the marriage is

irretrievably broken and an affidavit has been filed alleging that the parties have lived separate

and apart for a period of at least one year . . . .” Irretrievable breakdown is defined as

estrangement due to marital difficulties with no reasonable prospect of reconciliation. 23 Pa.

21

Page 26: JULY 2018 PENNSYLVANIA BAR EXAMINATION · JULY 2018 PENNSYLVANIA BAR EXAMINATION . Essay Questions and Examiners’ Analyses. and . Performance Test . Pennsylvania Board of Law Examiners

C.S.A. § 3103. Separate and apart is defined as cessation of cohabitation, whether living in the

same residence or not. In the event a complaint in divorce is filed and served, it shall be

presumed that the parties commenced to live separate and apart not later than the date that the

complaint was served. 23 Pa. C.S.A. § 3103. The period of separation commences when one

spouse clearly communicates his intent to dissolve the marital relationship to the other spouse.

Sinha v. Sinha, 526 A.2d 765, 767 (Pa. 1987). Since Ted explicitly notified Kayla on June 18,

2017, that he wanted a divorce, and had moved out, and they have been living separate and apart

for greater than one year as of July 7, 2018, it is likely that Ted could also pursue a no fault

ground for divorce under section 3301(d) under the Domestic Relations Code. 1

1 It is unlikely that Ted would be able to obtain a divorce based upon mutual consent under 23 Pa. C.S.A. § 3301(c)

as the facts expressly state that Kayla informed Ted that she would never agree to a divorce.

22

Page 27: JULY 2018 PENNSYLVANIA BAR EXAMINATION · JULY 2018 PENNSYLVANIA BAR EXAMINATION . Essay Questions and Examiners’ Analyses. and . Performance Test . Pennsylvania Board of Law Examiners

Question No. 3: Grading Guidelines

1. Criminal Law.

The applicants should discuss the elements of involuntary manslaughter and conclude that

the facts support the charge.

5 Points

2. Evidence

The applicants should identify prior bad act evidence and weigh its admission under the

probative value versus the prejudicial effect test in determining the admissibility of the

evidence.

5 Points

3. Criminal Procedure

The applicants should identify the issue as an inventory search exception to the search

warrant requirement and conclude that the purchase receipt would be admissible at trial.

6 Points

4. Family Law

The applicants should discuss whether Ted has grounds for a no-fault divorce under 3301(d)

based upon Ted and Kayla’s 1 year separation, and whether Ted has grounds for a fault

ground for divorce under 3301(a)(5) due to Kayla’s sentence of incarceration for a term in

excess of 2 years.

4 Points

23

Page 28: JULY 2018 PENNSYLVANIA BAR EXAMINATION · JULY 2018 PENNSYLVANIA BAR EXAMINATION . Essay Questions and Examiners’ Analyses. and . Performance Test . Pennsylvania Board of Law Examiners

Question No. 4

ManupPower (“MP”) employs 300 people and provides unarmed staffing for events all over the

country, such as concerts, sporting events, and political rallies. MP event personnel sell tickets the day of

events, collect tickets from attendees, provide seating information and directions, and monitor the

entrance and exit of attendees during events to identify “seat jumpers” trying to sit in non-assigned seats.

Ninety-five percent of the events staffed by MP also have an armed public police presence. Pursuant to

written policy, MP event personnel may not pursue or detain anyone and must contact public law

enforcement in the event of any physical aggression or resistance from event attendees.

In 2013, and on a whim, MP’s fitness-obsessed CEO implemented a physical fitness test that he

created from his own daily workouts as part of the final step of the hiring process for MP event personnel.

After interviews and background checks, candidates for event personnel positions were required to

complete various calisthenics and a one-mile run within a specified time period (the “Fitness Test”). If an

applicant passed the Fitness Test, that applicant was guaranteed an offer of employment as MP event

personnel. However, if an applicant failed the Fitness Test, then that applicant was not offered the job.

MP’s stated reason for using the Fitness Test in hiring decisions was so that MP clients could be

confident they were hiring “strong, tough, qualified event personnel.”

Between 2013 and the present, 400 applicants for security jobs made it to the final round of the

hiring process and took the Fitness Test. Half of those 400 applicants were men and half were women.

During those years, 90% of the 200 men administered the Fitness Test passed it (180 men total) and were

offered employment. However, only 5% of the 200 women who took the Fitness Test during those years

passed it (10 women total) and were offered employment. Betsy Smith, Libby Umber, and Liz Ellis

(collectively, “Plaintiffs”) all applied for MP security personnel jobs in 2016. Plaintiffs all excelled in

their interviews and passed their background checks. However, they failed the Fitness Test. Therefore,

none of the Plaintiffs was offered a position at MP.

After being denied employment at MP, Plaintiffs retained Big Law, a national plaintiffs’

employment law firm that has litigated hundreds of class action employment discrimination lawsuits.

24

Page 29: JULY 2018 PENNSYLVANIA BAR EXAMINATION · JULY 2018 PENNSYLVANIA BAR EXAMINATION . Essay Questions and Examiners’ Analyses. and . Performance Test . Pennsylvania Board of Law Examiners

Plaintiffs have exhausted their administrative remedies, and they have properly filed a lawsuit in the

appropriate federal district court against MP for alleged violations of Title VII of the Civil Rights Act of

1964 (the “Lawsuit”). Plaintiffs all want to reapply for jobs with MP in the future. Plaintiffs’ complaint

seeks declaratory and injunctive relief. Specifically, Plaintiffs want the court to declare that MP’s use of

the Fitness Test in making hiring decisions unlawful under Title VII and enjoin MP from using the

Fitness Test in connection with hiring decisions in the future. It is undisputed that there are at least 190

women who have failed the Fitness Test and not received offers of employment since MP starting using

this practice in making hiring decisions.

1. Plaintiffs’ complaint does not allege that MP engaged in intentional discrimination or that

the Fitness Test is facially discriminatory. Rather, Plaintiffs allege that MP’s practice of

using the Fitness Test as part of its hiring procedures results in an outcome that violates

Title VII. Based only on the facts set forth above, what theory of Title VII liability

should Plaintiffs assert against MP and with what likelihood of success?

2. Plaintiffs request that the federal district court certify a class of litigants comprised of “all

female applicants from the year 2013 to the present who were not offered MP event

personnel jobs based on their failure to pass the Fitness Test.” Plaintiffs seek to be the

class representative. What prerequisites will Plaintiffs have to establish to obtain class

certification and with what likelihood of success?

3. Assume for this question only that Plaintiffs did not seek class certification and

proceeded with only their individual claims. Over the past 16 months, MP has defended

the legality of the Fitness Test and vigorously litigated the Lawsuit, conducting countless

depositions, filing numerous motions, and producing thousands of pages of written

discovery. After the close of discovery and a month prior to the summary-judgment

deadline, Big Law received correspondence from MP’s attorney notifying Plaintiffs that

(1) effectively immediately, MP has stopped using the Fitness Test in making hiring

decisions, and (2) MP intends to seek dismissal of the Lawsuit because Plaintiffs have

requested only declaratory and injunctive relief related to the Fitness Test, and MP is no

longer using the Fitness Test to make hiring decisions. What federal constitutional

doctrine should MP raise in seeking dismissal of the Lawsuit and with what likely result?

25

Page 30: JULY 2018 PENNSYLVANIA BAR EXAMINATION · JULY 2018 PENNSYLVANIA BAR EXAMINATION . Essay Questions and Examiners’ Analyses. and . Performance Test . Pennsylvania Board of Law Examiners

Question No. 4: Examiner’s Analysis

1. Plaintiffs should assert a disparate impact claim against MP and are likely to succeed

Title VII of the Civil Rights Act of 1964 prohibits employment discrimination based on race,

color, religion, sex or national origin. 42 U.S.C. § 2000e-2(a)(1). “Title VII prohibits both intentional

discrimination (known as ‘disparate treatment’) as well as, in some cases, practices that are not

intended to discriminate but in fact have a disproportionately adverse effect on minorities

(known as ‘disparate impact’).” Ricci v. DeStefano, 557 U.S. 557, 577 (2009). In Griggs v. Duke Power Co., 401 U.S. 424, 431 (1971), the Supreme Court first interpreted Title VII to

prohibit the use of facially neutral practices that are “discriminatory in operation.” Since Griggs,

Title VII has been amended to codify the relevant burdens of proof in disparate impact cases:

An unlawful employment practice based on disparate impact is established . . . if . . . a

complaining party demonstrates that a respondent uses a particular employment practice

that causes a disparate impact on the basis of [a class protected by Title VII] and the

respondent fails to demonstrate that the challenged practice is job related for the position

in question and consistent with business necessity . . . .

42 U.S.C. § 2000e-2 (k)(1)(A)(i) (emphasis added). If an employer demonstrates that a

challenged practice is “job related for the position in question and consistent with business

necessity,” a plaintiff may still prevail in a Title VII disparate impact case “by showing that the

employer refuses to adopt an available alternative employment practice that has less disparate

impact and serves the employer’s legitimate needs.” Ricci, 557 U.S. at 578 (citing 42 U.S.C. §

2000e-2 (k)(1)(A)(i), (ii)).

Disparate Impact

[T]he plaintiff [in a disparate impact case] must offer statistical evidence of a kind and

degree sufficient to show that the practice in question has caused the exclusion of

applicants for jobs or promotions because of their membership in a protected group. Our

formulations, which have never been framed in terms of any rigid mathematical formula,

have consistently stressed that statistical disparities must be sufficiently substantial that

they raise such an inference of causation.

Watson v. Fort Worth Bank & Trust, 487 U.S. 977, 994-95 (1988) superseded by statute on other grounds, 42 U.S.C. 2000e-2(k)(1)(A).

The Equal Employment Opportunity Commission (EEOC) has issued guidelines stating

that “[a] selection rate for any race, sex, or ethnic group which is greater than four-fifths (4/5) (or

eighty percent) of the rate for the group with the highest rate will generally be regarded . . . as

evidence of adverse impact . . . .” 29 C.F.R. 1607.4(D). However, this is not a bright line

evidentiary rule. The regulation itself states that both smaller differences in selections rates may,

under some circumstances, establish disparate impact, and that greater differences may not in

other circumstances. Id. The Supreme Court has framed the question of the disparate impact

inquiry in terms such as whether challenged practices “operated to disqualify [a protected class]

26

Page 31: JULY 2018 PENNSYLVANIA BAR EXAMINATION · JULY 2018 PENNSYLVANIA BAR EXAMINATION . Essay Questions and Examiners’ Analyses. and . Performance Test . Pennsylvania Board of Law Examiners

at a substantially higher rate” than a non-protected class; Griggs, 401 U.S. at 426; whether tests

selected applicants “in a racial pattern significantly different from that of the pool of

applicants[,]” Albemarle Paper Co. v. Moody, 422 U.S. 405, 425 (1975); whether the employer

utilized “hiring and promotion practices disqualifying substantially disproportionate numbers of

blacks[,]” Washington v. Davis, 426 U.S. 229, 246-247 (1976); or whether there are standards

that “select applicants for hire in a significantly discriminatory pattern.” Dothard v. Rawlinson,

433 U.S. 321, 329 (1977).

Whether applying the EEOC 4/5 rule or the standards set forth in Supreme Court

precedent, Plaintiffs here can establish that the Fitness Test results in a disparate impact on

female applicants. Again, the selection rate for men under the MP Fitness Test is 90%, while the

selection rate for women is 5%. Because 90% of men in the final pool of applicants were hired,

72% of women (or 80% of 90%) from the same size pool must be hired in order to satisfy the 4/5

rule.1 However, only 5%, or 10 women, passed the Fitness Test and received offers of

employment. 5% is only 4.5% of 90%.2 This is woefully short of the 80% necessary to satisfy

the 4/5 rule. Moreover, the huge disparity between the number of men and women hired solely

because of performance on the Fitness Test evidences that the challenged practice (use of the

Fitness Test) “operated to disqualify [women] at a substantially higher rate” than it did men.

Griggs, 401 U.S. at 426. Thus, Plaintiffs here can easily establish their initial burden of showing

that the challenged practice – use of the Fitness Test as a job prerequisite – has a disparate

impact on women.

Business Necessity

Because the Fitness Test has a disparate impact on women, the burden will shift to MP to

show that the Fitness Test is “job related for the position in question and consistent with business

necessity.” 42 U.S.C. § 2000e-2(k)(1)(A)(i). In Griggs, the Supreme Court held that a

defendant could overcome a disparate impact claim by showing that employment tests that have

a discriminatory effect “bear a demonstrable relationship to successful performance of the jobs

for which it was used” and that “any given requirement must have a manifest relationship to the

employment in question.” Id. at 431, 432. In Albemarle Paper Co. v. Moody, 422 U.S. 405

(1975), the Supreme Court adopted EEOC guidance and held that test results must predict or

correlate with “important elements of work behavior which comprise or are relevant to the job or

jobs for which candidates are being evaluated.” 422 U.S. at 431 (quoting 29 C.F.R. § 1607.4(c)).

The Dothard Court stated that discriminatory employment requirements must “be shown to be

necessary to safe and efficient job performance.” Dothard, 433 U.S. at 331 n.14. In Dothard,

the employer argued that height and weight requirements for prison guards that had a

discriminatory impact on women were a business necessity because strength is an essential

quality for corrections officers. The Court noted that the employer had failed to identify the

specific amount of strength necessary to perform the job and had not demonstrated a relationship

between strength and its height and weight requirements. Id. at 331-32.

1 90 x .8 = 72. 2 90 x .05 = 4.5.

27

Page 32: JULY 2018 PENNSYLVANIA BAR EXAMINATION · JULY 2018 PENNSYLVANIA BAR EXAMINATION . Essay Questions and Examiners’ Analyses. and . Performance Test . Pennsylvania Board of Law Examiners

In all of the business necessity defense cases discussed above, the Supreme Court

required some evidence of the challenged test as it related to job performance. Employers must

provide evidence that the challenged practice “measure[s] the person for the job and not the

person in the abstract.” Dothard, 433 U.S. at 332 (quotation omitted). See also, EEOC v. Dial Corp., 469 F.3d 735, 742 (8th Cir. 2006) (stating that a validity study3 of employment tests “is

not necessary if the employer demonstrates the procedure is sufficiently related to safe and

efficient job performance”); Lanning v. Southeastern Pa. Transp. Auth., 181 F.3d 478, 490 (3d

Cir. 1999) (asking whether a cutoff score on physical fitness test for transit officers measures

“the minimum qualifications necessary for the successful performance of the job in question”).

Here, there are no facts to indicate that the Fitness Test given to event personnel job

applicants at MP are in any way related to predicted job performance. MP’s only proffered

reason for administering the Fitness Test is so that MP’s customers will know they are getting

“strong, tough, qualified event personnel.” Yet, the actual job duties do not require MP event

personnel to be “strong” or “tough.” Essentially, these employees are ticket-takers and ushers at

events. In fact, if an event attendee becomes physically aggressive or resistant, MP event

personnel are prohibited from detaining or pursuing such attendees and are required to contact

law enforcement. Thus, there seems no reason to test candidates’ physical strength or

cardiovascular endurance, and MP is unlikely to be successful in asserting the “business

necessity” defense to Plaintiffs’ disparate impact claim.

Less Discriminatory Alternative

Typically, if an employer can establish the business necessity defense, a plaintiff may

still succeed in a disparate impact case where he or she can establish the existence of a less

discriminatory alternative device. If, however, an employer fails to demonstrate that the

challenged practice or screening device is a business necessity, the plaintiff is not “required to

show the absence of a nondiscriminatory alternative.” Dial Corp., 469 F.3d at 743. As set forth

above, there are no facts whatsoever to support the contention that the Fitness Test is “job related

for the position in question and a business necessity.” Therefore, Plaintiffs will not have the

burden of establishing the existence of a less discriminatory alternative to the Fitness Test.

Because Plaintiffs will be able to present statistical evidence that the Fitness Test

disproportionately affects female applicants, and MP will be unable to satisfy its burden to show

that the Fitness Test is job related for the position in question and consistent with a business

necessity, Plaintiffs are likely to be successful in their disparate impact claim under Title VII.

2. The district court will apply the requirements of federal rule 23 and likely certify

the class.

A class action lawsuit is “‘an exception to the usual rule that litigation is conducted by

and on behalf of the individual named parties only.’” Wal-Mart Stores, Inc. v. Dukes, 564 U.S.

338, 348 (2011) (quoting Califano v. Yamasaki, 442 U. S. 682, 700-01 (1979)). Federal Rule of

3 In 1978, the EEOC adopted Uniform Guidelines on Employee Selection Procedures (“UGESP”), providing

employers with guidance on how to determine if a test or selection procedure is lawful when analyzing alleged

disparate impact claims under Title VII. 29 C.F.R. Part 1607.

28

Page 33: JULY 2018 PENNSYLVANIA BAR EXAMINATION · JULY 2018 PENNSYLVANIA BAR EXAMINATION . Essay Questions and Examiners’ Analyses. and . Performance Test . Pennsylvania Board of Law Examiners

Civil Procedure 23(a) sets forth four requirements - commonly referred to as numerosity,

commonality, typicality, and adequate representation – designed to ensure that a named plaintiff

is an appropriate class representative. Id. at 349 (noting that the prerequisites of Federal Rule

23(a) “effectively limit the class claims to those fairly encompassed by the named plaintiff’s

claims”) (quotation and citation omitted)).

Specifically, Federal Rule 23(a) provides:

(a) Prerequisites. One or more members of a class may sue or be sued as

representative parties on behalf of all members only if:

(1) the class is so numerous that joinder of all members is impracticable;

(2) there are questions of law or fact common to the class;

(3) the claims or defenses of the representative parties are typical of the

claims or defenses of the class; and

(4) the representative parties will fairly and adequately protect the interests of the

class.

F.R.C.P. 23(a) (emphasis added).

In addition to satisfying all four requirements of Federal Rule 23(a), a putative class

representative must also establish that the putative class meets at least one of the three criteria of

Federal Rule 23(b). Wal-Mart, 564 U.S. at 345. Federal Rule 23(b) provides as follows:

(b) Types of Class Actions. A class action may be maintained if Rule 23(a) is satisfied

and if:

(1) prosecuting separate actions by or against individual class members would

create a risk of:

(A) inconsistent or varying adjudications with respect to individual

class members that would establish incompatible standards

of conduct for the party opposing the class; or

(B) adjudications with respect to individual class members that, as a

practical matter, would be dispositive of the interests of the

other members not parties to the individual adjudications or

would substantially impair or impede their ability to protect

their interests;

(2) the party opposing the class has acted or refused to act on grounds that

29

Page 34: JULY 2018 PENNSYLVANIA BAR EXAMINATION · JULY 2018 PENNSYLVANIA BAR EXAMINATION . Essay Questions and Examiners’ Analyses. and . Performance Test . Pennsylvania Board of Law Examiners

apply generally to the class, so that final injunctive relief or

corresponding declaratory relief is appropriate respecting the class

as a whole; or

(3) the court finds that the questions of law or fact common to the class

members predominate over any questions affecting only individual

members . . . .

F.R.C.P. 23(b).

Numerosity

Here, the representative plaintiffs seek certification of a class of approximately 190

women who have been denied employment at MP because they did not pass the Fitness Test

(200 applicants less the 5%, or 10 women, who were offered jobs). It is almost certain that the

court will hold the proposed class-size is sufficient to meet the numerosity requirement set forth

in Federal Rule 23(a)(1). “No minimum number of plaintiffs is required to maintain a suit as a

class action, but generally if the named plaintiff demonstrates that the potential number of

plaintiffs exceeds 40, the first prong of Rule 23(a) has been met.” Stewart v. Abraham, 275 F.3d

220, 226-27 (3d Cir. 2001) (citing 5 James Wm. Moore et al., Moore’s Federal Practice §

23.22[3][a] (Matthew Bender 3d ed. 1999)).

Commonality and Typicality

The requirements of commonality and typicality both “seek to assure that the action can

be practically and efficiently maintained and that the interests of the absentees will be fairly and

adequately represented.” Baby Neal for and by Kanter v. Casey, 43 F.3d 48, 56 (3d Cir. 1994).

“[C]ommonality requires the plaintiff to demonstrate that the class members ‘have suffered the

same injury.’” Wal-Mart, 564 U.S. at 349-50 (quoting General Telephone Co. of the Southwest v. Falcon, 457 U.S. 147, 157 (1982). Stated another way, for Rule 23(a)(2) to be satisfied,

claims must “depend upon a common contention [that is] . . . of such a nature that it is capable of

classwide resolution – which means that determination of its truth or falsity will resolve an issue

that is central to the validity of each one of the claims in one stroke.” Id. at 350. In Wal-Mart, a

plurality of the Supreme Court held that there was no commonality of claims within a putative

class of 1.5 million female employees of Wal-Mart because the challenged pay and promotion

decisions were the result of discretionary decisions made by local supervisors rather than arising

out of a company-wide policy or practice. Id. at 352-60.

“The typicality inquiry is intended to assess whether the action can be efficiently

maintained as a class and whether the named plaintiffs have incentives that align with those of

absent class members so as to assure that the absentees' interests will be fairly represented.”

Baby Neal, 43 F.3d at 57 (citing 3B JAMES W. MOORE & John E. Kennedy, MOORE’S FEDERAL

PRACTICE ¶ 23.06-02 (1993); 1 H. NEWBURG & A. Conte, NEWBERG ON CLASS ACTIONS § 3.13

(1992)). The purpose of the typicality requirement is “to preclude certification of those cases

where the legal theories of the named plaintiffs potentially conflict with those of the absentees . .

. .” Id. (citation omitted).

30

Page 35: JULY 2018 PENNSYLVANIA BAR EXAMINATION · JULY 2018 PENNSYLVANIA BAR EXAMINATION . Essay Questions and Examiners’ Analyses. and . Performance Test . Pennsylvania Board of Law Examiners

While commonality and typicality are separate requirements for class certification, the

Supreme Court merged them together in the disparate impact context in holding the District

Court erred in certifying a class because the class representative failed to satisfy the commonality

and typicality requirements of Rule 23(a)(1) and (a)(2) in a Title VII action. Falcon, 457 U.S. at

158. In Falcon, the Court stated a “wide gap” existed between an individual’s claim of

discrimination in a promotion decision and the existence of a class of individuals who share the

same injury as that individual. Id. at 157. The Court explained that specific questions of law or

fact “common to the claims of [the class representative] and of the members of the class he

sought to represent” were necessary for a court to certify a class. Id. at 158. To that end, the

Falcon court indicated that “a class action on behalf of every applicant or employee who might

have been prejudiced by [a biased testing procedure] clearly would satisfy the commonality

and typicality requirements of Rule 23(a).” Id. at 159 n. 15 (emphasis added).

At issue here is MP’s use of the Fitness Test to determine whether to offer event

personnel jobs to applicants. Every member of the putative class applied for an MP event

personnel position, successfully interviewed, and passed a background check. However, each

was also denied an offer of employment because she did not pass the Fitness Test. This scenario

is clearly different than those in Wal-Mart and Falcon where the Supreme Court did not find

commonality to be present. Rather, it involves a biased testing procedure applied evenly across

all candidates for the job. As the Supreme Court said in Falcon, this “clearly would satisfy the

commonality and typicality requirements of Rule 23(a).” Id.

Adequate Representation

Finally, Plaintiffs must adequately represent the other members of the putative class.

“The adequacy inquiry under Rule 23(a)(4) serves to uncover conflicts of interest between

named parties and the class they seek to represent.” Amchem Prods., Inc. v. Windsor, 521 U.S.

591, 625 (1997). To that end, a class representative must have the same interest and suffer the

same injury as the class members she seeks to represent. Id. at 626 (citations and quotations

omitted). “Adequate representation depends on two factors: (a) the plaintiff’s attorney must be

qualified, experienced, and generally able to conduct the proposed litigation, and (b) the plaintiff

must not have interests antagonistic to those of the class.” New Directions Treatment Servs. v. City of Reading, 490 F.3d 293, 313 (3d Cir. 2007) (quotation and citation omitted). See also, In re Drexel Burnham Lambert Group, 960 F.2d 285, 291 (2d Cir. 1992).

Here, all of the members of the putative class are identically situated – they were denied

an offer of employment at MP because of their inability to pass the Fitness Test. Thus, there is

no basis on which the court could conclude that the named plaintiffs have interests antagonistic

to the class members they seek to represent. Moreover, Plaintiffs have retained a national

plaintiffs’ employment law firm that has successfully certified classes in hundreds of

employment discrimination cases. There are no facts that would suggest that plaintiff’s counsel

is not “qualified, experienced, and generally able to conduct the proposed litigation.” Id.

Thus, Plaintiffs would likely be able to meet all of the requirements for class certification

set forth in Rule 23(a).

31

Page 36: JULY 2018 PENNSYLVANIA BAR EXAMINATION · JULY 2018 PENNSYLVANIA BAR EXAMINATION . Essay Questions and Examiners’ Analyses. and . Performance Test . Pennsylvania Board of Law Examiners

Rule 23(b)

Once the requirements of Federal Rule 23(a) are determined to have been met, the court

must examine whether at least one of the criteria in Rule 23(b) is satisfied. F.R.C.P. 23(b). Here,

the facts support a finding that Plaintiffs can satisfy Rule 23(b)(2), which requires that: “the

party opposing the class has acted or refused to act on grounds that apply generally to the class,

so that final injunctive relief or corresponding declaratory relief is appropriate respecting the

class as a whole[.]” F.R.C.P. 23(b)(2).

Here, Plaintiffs’ allegations are that MP (the party opposing the class) improperly utilized

the Fitness Test in deciding whether to make offers of employment (i.e., acted or refused to act),

and that every member of the putative class was denied an offer of employment as a result (i.e.,

grounds that apply to the class). Plaintiffs are requesting identical injunctive relief that is

appropriate to the class as whole (i.e., an injunction against utilizing the Fitness Test in future

hiring decisions).

Because Plaintiffs can meet all of the requirements of Federal Rule 23(a) and the criteria

set forth in Rule 23(b)(2), it is likely that the district court will certify a class of all women

denied offers of employment as MP event personnel in the past four years based on their inability

to complete the Fitness Test.

3. MP should argue that plaintiffs’ claims are moot and plaintiffs should respond that

voluntary cessation of allegedly unlawful conduct is insufficient to moot their claims.

If MP is intent on trying to have the Lawsuit dismissed at this late stage of the litigation,

it should raise the doctrine of mootness, arguing that there is no longer a live “case or

controversy” because MP is no longer administering or using the Fitness Test in connection with

hiring decisions.

Article III of the United States Constitution limits the judicial power of federal courts to

“cases or controversies.” Friends of the Earth, Inc. v. Laidlaw Envtl. Servs., Inc., 528 U.S. 167,

180 (2000) (“The Constitution’s case-or-controversy limitation on federal judicial authority, Art.

III, § 2, underpins both our standing and our mootness jurisprudence . . .”). “‘Simply stated, a

case is moot when the issues presented are no longer ‘live’ or the parties lack a legally

cognizable interest in the outcome.’” Los Angeles Cty. v. Davis, 440 U.S. 625, 631 (1979)

(quoting Powell v. McCormack, 395 U.S. 486, 496 (1969)).

At first glance, MP’s decision to stop utilizing the Fitness Test would indicate that its

claims are moot because Plaintiffs’ complaint requests only declaratory and injunctive relief

related to MP’s use of the test in hiring decisions. However, Plaintiffs should respond to any

effort of MP to get the case dismissed on the basis of mootness by raising one of the “long-

recognized exceptions” to the doctrine, specifically, the exception of “voluntary cessation” of an

allegedly unlawful practice. Laidlaw, 528 U.S. at 189-90.

It is well settled that a defendant’s voluntary cessation of a

challenged practice does not deprive a federal court of its power to

32

Page 37: JULY 2018 PENNSYLVANIA BAR EXAMINATION · JULY 2018 PENNSYLVANIA BAR EXAMINATION . Essay Questions and Examiners’ Analyses. and . Performance Test . Pennsylvania Board of Law Examiners

determine the legality of the practice. If it did, the courts would be

compelled to leave the defendant . . . free to return to his old ways.

In accordance with this principle, the standard we have announced

for determining whether a case has been mooted by the defendant’s

voluntary conduct is stringent: A case might become moot if

subsequent events made it absolutely clear that the allegedly

wrongful behavior could not reasonably be expected to recur. The

heavy burden of persuading the court that the challenged conduct

cannot reasonably be expected to start up again lies with the party

asserting mootness.

Id. (citations and quotations omitted).

One factor that the Supreme Court has looked at to determine whether voluntary

cessation renders a claim moot or is likely to start again in the future is whether or not the

defendant has defended its actions. Parents Involved in Cmty. Schs. v. Seattle Sch. Dist. No. 1,

551 U.S. 701, 719 (2007) (rejecting a school district’s mootness argument where the district

ceased utilizing a challenged race-based policy during the course of litigation, in part because

“the district vigorously defends the constitutionality of its race-based program, and nowhere

suggests that if this litigation is resolved it its favor it will not resume using race to assign

students.”); see also, DeJohn v. Temple Univ., 537 F.3d 301, 304, 310-11 (3rd Cir. 2008)

(rejecting mootness argument where the defendant university modified a challenged policy three

weeks prior to the dispositive motion deadline but had defended its policy throughout litigation;

did not change it until discovery was almost over; and there was no indication that the university

had deliberated over its decision in such a way to indicate that it would not “revert back to its old

policy” in the future).

As in the above cases, there are no facts to suggest that MP would not return to using the

Fitness Test in connection with hiring decisions in the future. MP vigorously defended the

legality of the Fitness Test for more than a year and litigated the case extensively. It was only at

the eleventh hour, a few weeks before summary judgment motions were due, that MP ceased the

challenged practice. Simultaneously, MP indicated that it was going to seek dismissal of the

case.

In sum, there are no facts upon which the court would conclude that MP will not restart

the challenged practice. Accordingly, the court is unlikely to dismiss the Lawsuit.

33

Page 38: JULY 2018 PENNSYLVANIA BAR EXAMINATION · JULY 2018 PENNSYLVANIA BAR EXAMINATION . Essay Questions and Examiners’ Analyses. and . Performance Test . Pennsylvania Board of Law Examiners

Question No. 4: Grading Guidelines

1. Employment Law

Applicants should demonstrate knowledge of the relative burdens of proof for disparate impact claims

under Title VII and apply a set of facts to these elements to reach a well-reasoned conclusion.

8 points

2. Civil Procedure

Applicants should demonstrate knowledge of the prerequisites for class certification contained in Federal

Rule of Civil Procedure 23 and apply a set of facts to these criteria to reach a well-reasoned conclusion.

8 points

3. Constitutional Law

Applicants should identify mootness as a possible basis for dismissal under the facts presented and exhibit

knowledge of the voluntary cessation exception to the mootness doctrine to reach a well-reasoned

conclusion.

4 points

34

Page 39: JULY 2018 PENNSYLVANIA BAR EXAMINATION · JULY 2018 PENNSYLVANIA BAR EXAMINATION . Essay Questions and Examiners’ Analyses. and . Performance Test . Pennsylvania Board of Law Examiners

Question No. 5

Owner subdivided his farmland near Blueberry Hill, Pennsylvania, into two large parcels,

Lot A and Lot B, pursuant to a recorded subdivision plan (“the Plan”). The Plan included the

following declaration (the “Declaration”): “Alcoholic beverages shall not be made, sold, or

consumed on any parcels in this Plan. The benefits and burdens of this restriction shall be

applicable to all grantees, their heirs, successors, and assigns.” Owner eventually conveyed Lots

A and B to separate grantees by valid deeds that expressly referred to the Declaration. After

several succeeding valid transfers over an extended period of years, Lot A came to be owned by

KBL, a national cable television sports channel, which used the building previously constructed

on Lot A as its broadcast studios. Lot B came to be owned by Gatebriar, which operated an in-

patient drug and alcohol treatment facility on the property. The deed conveying Lot A to KBL

did not mention the Declaration. The deed conveying Lot B to Gatebriar stated that the

conveyance was “subject to all restrictions of record.”

KBL hired Al, who lived in Big City, Pennsylvania, for its most prestigious on-air job:

doing play-by-play for KBL’s nationally broadcast Sunday night pro-football games. Al’s

contract with KBL was for two years at $750,000 per year terminable only for cause. After

signing his KBL contract, Al became engaged to marry Brenda, his long-time agent, who

handled every aspect of Al’s broadcast career. Before marrying Al, Brenda conveyed Blackacre,

her Big City home, to her brothers, Chuck and Dave, using a deed form that she found on the

Internet. The deed stated in part: “Said Grantor does grant, bargain, sell, alien and enfeoff

Blackacre JOINTLY unto the said Grantees and their heirs forever.”

Following their marriage, Al and Brenda purchased a home in Blueberry Hill known as

Whiteacre. The deed validly conveying Whiteacre simply described Al and Brenda as the

35

Page 40: JULY 2018 PENNSYLVANIA BAR EXAMINATION · JULY 2018 PENNSYLVANIA BAR EXAMINATION . Essay Questions and Examiners’ Analyses. and . Performance Test . Pennsylvania Board of Law Examiners

grantees, and not as husband and wife.

Prior to the start of the second year of Al’s broadcasting contract, KBL materially

breached by firing Al without cause. Despite her best efforts, Brenda only was able to obtain an

offer from a low-watt Blueberry Hill radio station to have Al do play-by-play for local high

school football games for $25,000. Al, who also participated in the search for a new job,

rejected this offer and spent his time playing on-line “Fantasy Football.” In the process, Al ran

up a large personal debt on his Big Bank credit card that he was unable to pay.

KBL contacted What-A-Mess (“Mess”), a local contractor, about remodeling one of its

studios. Mess stated, “I’ll do the job for $200,000. You have 30 days to respond.” KBL’s

authorized representative replied, “Our budget is a little tight right now. We’ll keep your offer

under advisement, but if you wish to close the matter at once we’ll pay you $125,000.” Mess did

not respond to KBL’s statement.

1. Before the end of the 30 days, KBL’s representative called Mess and stated, “We

found some extra money in our budget. KBL accepts your offer to remodel its

studio sets for $200,000.” Was a valid contract formed between KBL and Mess?

2 (a). One year after Brenda conveyed Blackacre to Chuck and Dave, Chuck suddenly

died. By will, Chuck left his entire estate to the Human Fund, a Big City charity.

What is the state of title to Blackacre following Chuck’s death?

(b). When Al did not pay his huge credit card bill, Big Bank sued and obtained a

judgment solely against Al. To recover on this judgment, Big Bank wants to

force a sale of Whiteacre. Will Big Bank succeed in forcing a sale of Whiteacre?

3. Al sued KBL for breach of contract. What contract law argument should KBL

raise to preclude or reduce Al’s damages for material breach of his contract and

with what result?

4. Assume for purposes of this question only that Mess and KBL entered into a new

contract to convert part of KBL’s studios into a sports bar/brew pub called

“KBL’s Out.” What relief should Gatebriar seek to stop the construction of the

sports bar/brew pub on Lot A and what legal theory, if any, supports its right to

relief?

36

Page 41: JULY 2018 PENNSYLVANIA BAR EXAMINATION · JULY 2018 PENNSYLVANIA BAR EXAMINATION . Essay Questions and Examiners’ Analyses. and . Performance Test . Pennsylvania Board of Law Examiners

Question No. 5: Examiner’s Analysis

1. A valid contract was created between KBL and Mess because Mess’s offer was not

rejected by KBL’s counter-offer. KBL’s response clearly manifested an intention

that its counter-offer should not be understood as a rejection of Mess’s offer.

To create a contract under Pennsylvania law, “there must be an offer on one side and an

acceptance on the other side.” Cawthorne v. Erie Ins. Grp., 782 A.2d 1037, 1038 (Pa. Super.

2001).

The Restatement (Second) of Contracts describes an offer as “a manifestation of

willingness to enter into a bargain, so made as to justify another person in understanding that his

assent to that bargain is invited and will conclude it.” RESTATEMENT (SECOND) OF CONTRACTS

§ 24 (Am. Law Inst. 1981); accord, Philadelphia Newspapers, Inc. v. Unemployment Comp. Bd. of Review, 426 A.2d 1289, 1290 n.3 (Pa. Cmwlth. 1981). “It is basic contract law that one

cannot suppose, believe, suspect, imagine or hope that an offer has been made. An offer must be

intentional, definite, [sic] in its terms and communicated, otherwise the minds cannot meet. . . .

An offer must define its terms, specify the thing offered and be an intention of the present or the

future to be bound.” Morosetti v. Louisiana Land and Exploration Co., 564 A.2d 151, 152 (Pa.

1989) (footnotes omitted). “Whether a statement is intended as an offer must be examined in

light of the surrounding circumstances.” Reed v. Pittsburgh Bd. of Pub. Educ., 862 A.2d 131,

135 (Pa. Cmwlth. 2004). Some of the circumstances relevant to the determination of whether an

offer is made “‘include the terms of any previous inquiry, the completeness of the terms of the

suggested bargain, and the number of persons to whom a communication is addressed.’” Beaver Valley Alloy Foundry, Co. v. Therma–Fab, Inc., 814 A.2d 217, 222 (Pa. Super. 2002), quoting

RESTATEMENT (SECOND) OF CONTRACTS, § 26 cmt. c.

In this case, Mess stated, “I’ll do the job for $200,000.” Mess’s statement clearly

manifested an intent to be bound under definite terms and would be viewed as an offer to KBL.

“An offer creates a power of acceptance in a specified offeree to transform the offeror’s

promise into a contractual obligation.” Philadelphia Newspapers, Inc., 426 A.2d at 1290 n.3. “It

is a basic principle of the law of contracts that an acceptance must be unconditional and

absolute.” O’Brien v. Nationwide Mut. Ins. Co., 689 A.2d 254, 258 (Pa. Super. 1997) (quotation

and citation omitted). However, “a reply to an offer which purports to accept it, but changes the

conditions of the offer, is not an acceptance, but is a counter-offer.” First Home Sav. Bank, FSB v. Nernberg, 648 A.2d 9, 15 (Pa. Super. 1994), appeal denied, 657 A.2d 491 (Pa. 1995) (citations

omitted).

Section 39(1) of the Restatement (Second) of Contracts defines a counter-offer as “an

offer made by an offeree to his offeror relating to the same matter as the original offer and

proposing a substitute bargain differing from that proposed by the original offer.”

RESTATEMENT (SECOND) OF CONTRACTS § 39(1) (1981). A counter-offer normally has the

same effect as a rejection of an offer because it terminates the offeree’s power to accept the

original offer. First Home Savings Bank, 648 A.2d at 15; see also, RESTATEMENT (SECOND) OF

37

Page 42: JULY 2018 PENNSYLVANIA BAR EXAMINATION · JULY 2018 PENNSYLVANIA BAR EXAMINATION . Essay Questions and Examiners’ Analyses. and . Performance Test . Pennsylvania Board of Law Examiners

CONTRACTS § 39, cmt. a. (“It is often said that a counter-offer is a rejection, and it does have the

same effect in terminating the offeree's power of acceptance.”). KBL’s reply that “if you wish to

close the matter at once we’ll pay you $125,000” proposed a substitute bargain and would be

viewed as a counter-offer to Mess.

The normal operation of a counter-offer as a rejection of the original offer, however, can

be overcome by a clear manifestation of “a contrary intention” by an offeree. RESTATEMENT

(SECOND) OF CONTRACTS § 39(2). Although KBL’s authorized representative made a counter-

offer by proposing a new price for Mess to remodel the KBL studios, KBL also stated that it was

taking Mess’s original offer “under advisement.” By using this language, the offeree (KBL)

clearly manifested an intention to the offeror (Mess) that the counter-offer should not be

understood as a rejection of Mess’s original offer. Id., cmt. c; see also, id. § 38 (2) (“A

manifestation of intention not to accept an offer is a rejection unless the offeree manifests an

intention to take it under further advisement.”). Therefore, even though KBL made a counter-

offer, it did not reject Mess’s offer. Further, each party had a power of acceptance: Mess to

accept KBL’s counter-offer at $125,000 and KBL to accept Mess’s original offer at $200,000.

See, JOHN E. MURRAY, JR., MURRAY ON CONTRACTS, § 43[D] (5th ed. 2011).

By clearly manifesting an intention that its counter-offer to Mess should not be

understood as a rejection of Mess’s original offer, KBL still retained its power to accept Mess’s

offer within the specified time period.1 Because KBL accepted Mess’s offer to remodel the KBL

studios at a cost of $200,000 within the thirty-day period specified in the offer, a valid contract

was formed between KBL and Mess. See, RESTATEMENT (SECOND) OF CONTRACTS § 39(2),

Illustration 3.

2 (a). The language in Brenda’s deed will be construed as creating a tenancy in common

between Chuck and Dave. Upon Chuck’s death, Dave and the Human Fund each

will own undivided one-half interests in Blackacre as tenants in common.

Concurrent or joint estates are estates owned by two or more persons at the same time.

RALPH E. BOYER ET AL, THE LAW OF PROPERTY: AN INTRODUCTORY SURVEY, 100 (West

Publishing Co. 4th ed. 1991). There are three types of joint estates: tenancy in common, joint

tenancy, and tenancy by the entireties. JAN Z. KRASNOWIECKI, KRASNOWIECKI ON REAL

PROPERTY LAW AND PRACTICE, 137 (PBI Press 2nd ed. 2008). Of the three types of joint or

concurrent ownership of property, tenancy by the entireties can exist only between co-tenants

who are married. Id. at 156; Boyer, supra, at 103.

To create a joint tenancy, the so-called four “unities” of time, title, interest and

possession must be present. In re Estate of Quick, 905 A.2d 471, 474 (Pa.2006) (citation

omitted). The distinctive characteristic of a joint tenancy is the right of survivorship. Upon the

death of one joint tenant, the surviving joint tenant becomes the sole owner of the entire interest

1 As the master of the offer, the offeror may place whatever time limitations upon the exercise of the power to accept

the offer as the offeror deems desirable. MURRAY, supra § 42 (B). “If an offer prescribes the place, time or manner

of acceptance its terms in this respect must be complied with in order to create a contract.” RESTATEMENT

(SECOND) OF CONTRACTS § 60 (1981). See also, Id. § 41 (1), cmt. a. (An offer “may prescribe a time limit for

acceptance.”)

38

Page 43: JULY 2018 PENNSYLVANIA BAR EXAMINATION · JULY 2018 PENNSYLVANIA BAR EXAMINATION . Essay Questions and Examiners’ Analyses. and . Performance Test . Pennsylvania Board of Law Examiners

in the property. In re Parkhurst’s Estate, 167 A.2d 476, 478 (Pa. 1961). “The survivorship

characteristic of a joint tenancy precludes a joint tenant from disposing of his interest by will.”

General Credit Co. v. Cleck, 609 A.2d 553, 556 (Pa. Super. 1992), appeal discontinued, 613

A.2d 560 (Pa. 1992).

In contrast, “[a] tenancy in common is [a concurrent] estate in which there is unity of

possession but separate and distinct titles.” In re Sale of Property of Dalessio, 657 A.2d 1386,

1387 n.1 (Pa. Cmwlth. 1995). Unlike a joint tenancy, where joint tenants own an undivided part

of the whole, each tenant in common owns the whole of the undivided interest. 1 RONALD M.

FRIEDMAN, LADNER PENNSYLVANIA REAL ESTATE LAW, § 8.03 (6th ed. 2013). “It is basic

property law that ‘a right of survivorship’ is not associated with a ‘tenancy in common.’”

Margarite v. Ewald, 381 A.2d 480, 481 (Pa. Super. 1977). “In a tenancy in common, . . . when

one co-tenant dies, his interest descends or passes by will to his heirs or devisees; the remaining

co-tenants acquire no additional interest in such an estate.” Edel v. Edel, 424 A.2d 946, 948 (Pa.

Super. 1981) (citations omitted).

At early common law, joint tenancies were favored and a conveyance or devise to two or

more persons who were not husband and wife was presumed to create a joint tenancy with the

right of survivorship. Under contemporary law, joint tenancies with right of survivorship are in

disfavor and the presumption now is that all co-owners who are not husband and wife hold

jointly as tenants in common. In re Estate of Michael, 218 A.2d 338, 340-41 (Pa. 1966).

Pennsylvania law follows the trend of disfavoring joint tenancies with right of survivorship.

Pennsylvania Bank & Trust Co. v. Thompson, 247 A.2d 771 (Pa. 1968). By statute, the incident

of survivorship was taken away as a presumption and an instrument creating a joint estate is

presumed to create a tenancy in common. See, 68 P.S. § 110 (2004).

Nevertheless, Pennsylvania’s statute on joint tenancies has been held only to be a statute

of construction; it does not proscribe the creation of a joint tenancy with right of survivorship.

Teacher v. Kijurina, 76 A.2d 197, 201 (Pa. 1950). In light of Pennsylvania’s statutory

preference for tenancy in common, the question whether a particular conveyance or devise to

two or more persons creates a joint tenancy with right of survivorship becomes one of intent.

Maxwell v. Saylor, 58 A.2d 355, 356 (Pa.1948). In order to engraft the right of survivorship on a

co-tenancy which might otherwise be a tenancy in common, the intent to do so must be

expressed with sufficient clearness to overcome the statutory presumption that survivorship is

not intended. Isherwood v. Springs-First Nat’l Bank, 74 A.2d 89, 91 (Pa.1950).

In Kijurina, the Pennsylvania Supreme Court observed that the necessary intent to

overcome the statutory presumption against a joint tenancy with right of survivorship could be

discerned by the grantor’s use of words in the conveyance such as “as joint tenants and not as

tenants in common,” or “survivor,” or “with the right of survivorship.” Kijurina, 76 A.2d at 201.

(internal quotations and citations omitted). By contrast, in this case, the pre-printed language in

Brenda’s deed granting Blackacre to Chuck and Dave only used the word “jointly.” In the

absence of any language in the deed indicating an intention to create a joint tenancy with right of

survivorship, a court applying Pennsylvania law would probably hold that the statutory

presumption in favor of tenancy in common should prevail in this instance and that Chuck and

Dave owned Blackacre as tenants in common.

39

Page 44: JULY 2018 PENNSYLVANIA BAR EXAMINATION · JULY 2018 PENNSYLVANIA BAR EXAMINATION . Essay Questions and Examiners’ Analyses. and . Performance Test . Pennsylvania Board of Law Examiners

Because Chuck’s interest in Blackacre was owned as a tenant in common, he was able to

transfer that interest by will to the Human Fund. Therefore, Dave and the Human Fund each

own an undivided one-half interest in Blackacre as tenants in common.

2 (b). By virtue of their marital status, Al and Brenda own Whiteacre as tenants by the

entireties. Big Bank cannot recover its judgment against Al by forcing a sale of

Whiteacre as long as the tenancy by the entireties remains intact.

The deed conveying Whiteacre did not state that Al and Brenda were married. The

failure of the deed to note this fact, however, is inconsequential under Pennsylvania law because

the conveyance of real estate to two grantees who are husband and wife is presumed to create a

tenancy by the entireties. Holmes Estate, 200 A.2d 745, 747 (Pa.1964). It is the actual married

status of the grantees and not necessarily the words stated or omitted in the instrument that

determines their right to take as tenants by the entireties. 1 RONALD M. FRIEDMAN, LADNER

PENNSYLVANIA REAL ESTATE LAW, § 8.04 (b) (6th ed. 2013).

To overcome the presumption that a conveyance to grantees who are married creates a

tenancy by the entireties, there must be clear and convincing evidence of a contrary intent.

Holmes, 200 A.2d at 747, see also Brenner v. Sukenik, 189 A.2d 246, 249 (Pa.1963). In this

case, there are no facts that would indicate an intention to create a different estate in Whiteacre

between Al and Brenda. Therefore, the presumption would control and Al and Brenda would

own Blackacre as tenants by the entireties.

A tenancy by the entireties is predicated upon legal unity of husband and wife. Beihl v. Martin, 84 A. 953, 954 (Pa.1912). From the inception of the estate, “each spouse is seized of the

whole or the entirety and not of a share, moiety, or divisible part.” Gasner v. Pierce, 134 A. 494,

495 (Pa. 1926). Because a tenancy by the entireties is grounded in the conception of the estate as

a single indivisible unit, the law in Pennsylvania is quite clear on whether a creditor of one of the

tenants can enforce a debt against property held as a tenancy by the entireties:

[A] judgment creditor may execute on entireties property to enforce his judgment

if both spouses are joint debtors. However, if only one spouse is a debtor,

entireties property is immune from process, petition, levy, execution or sale. In

the latter situation, the judgment creditor has only a potential lien against property

held by the entireties based on the debtor spouse’s expectancy to become sole

owner.

ISN Bank v. Rajaratham, 83 A.3d 170, 174 (Pa. Super. 2014) (emphasis added, quotation

and citations omitted).

As long as Al and Brenda are both alive and married to one another, they will continue to

own Whiteacre as tenants by the entireties. Therefore, Big Bank’s attempt to recover Al’s

personal credit card debt by forcing a sale of Whiteacre will not be successful.

40

Page 45: JULY 2018 PENNSYLVANIA BAR EXAMINATION · JULY 2018 PENNSYLVANIA BAR EXAMINATION . Essay Questions and Examiners’ Analyses. and . Performance Test . Pennsylvania Board of Law Examiners

3. KBL should argue that Al’s damages should be reduced because he failed to make

reasonable efforts to mitigate his damages. The success of KBL’s argument depends

on whether Al’s rejection of the job doing radio play-by-play for high school

football games is considered to be reasonable.

“[I]t is a universally accepted rule that [an injured] promisee cannot recover those

damages for breach of contract which he could have avoided through the exercise of reasonable

diligence if he can do so without incurring undue risk, expense or humiliation.” JOHN E.

MURRAY, JR., MURRAY ON CONTRACTS, § 123 [A] (5th ed. 2011); see also, RESTATEMENT

(SECOND) OF CONTRACTS § 350 (1) (1981). The rule of mitigation or limitation of avoidable

losses is often referred to in Pennsylvania case law as a “duty” to mitigate damages. See, e.g., TruServ Corp. v. Morgan's Tool & Supply Co., Inc., 39 A.3d 253, 262 (Pa. 2012) (citations

omitted). A failure to reasonably attempt to mitigate damages reduces a party’s recovery by the

amount of loss that could have been avoided by reasonable mitigation. State Pub. Sch. Bldg. Auth. v. W.M. Anderson Co., 410 A.2d 1329, 1331 (Pa. Cmwlth. 1980); RESTATEMENT

(SECOND) OF CONTRACTS § 350, cmt. b.

In determining whether an injured party has acted appropriately to mitigate damages, the

test to be applied is one of reasonableness. Id., § 350 (2). Whether an injured party’s conduct in

response to a breach is reasonable is to be determined from all the facts and circumstances of

each case at the time in question. Schnabel Assoc., Inc. v. T & M Interiors, 507 A.2d 1241, 1243

(Pa. Super. 1986) citing Toyota Indus. Trucks U.S.A. Inc. v. Citizens Nat’l Bank of Evans City,

611 F.2d 465, 471 (3d Cir. 1979). The burden is on the breaching party to show how losses

could have been avoided through the efforts of the injured party. TruServ Corp., 39 A.3d at 262.

In a breach of employment contract case, “the measure of damages is the wages which

were to be paid less any amount actually earned or which might have been earned through the

exercise of reasonable diligence in seeking other similar employment.” Delliponti v. DeAngelis,

681 A.2d 1261, 1265 (Pa. 1996) (citation omitted). “An employee, however, need not accept

employment that is substantially different from the original employment to mitigate damages.”

MURRAY, supra § 123[D]. Thus, the burden is upon the breaching party to prove that other

substantially equivalent positions were available to the injured party and that the injured party

failed to use reasonable diligence in attempting to secure those positions. Delliponti, 681 A.2d at

1265.

Whether an injured party’s “substitute employment is ‘comparable,’ ‘substantially

similar,’ or ‘substantially equivalent’ to the original employment can be a difficult issue.”

MURRAY, supra § 123 [D]. In this case, Al had an offer to do play-by-play for high school

football games on a low-watt Blueberry Hill radio station for $25,000. Al’s original employment

was doing television play-by-play of KBL’s nationally broadcast professional football games, the

most prestigious on-air job with the network, at $750,000 per year. The trier of fact will have to

carefully consider all of these circumstances in determining whether the other employment

opportunity offered to Al was substantially similar to his previous position at KBL and thus

made his failure to take the new position with the Blueberry Hill radio station an unreasonable

effort to mitigate his damages due to KBL’s material breach of contract.

41

Page 46: JULY 2018 PENNSYLVANIA BAR EXAMINATION · JULY 2018 PENNSYLVANIA BAR EXAMINATION . Essay Questions and Examiners’ Analyses. and . Performance Test . Pennsylvania Board of Law Examiners

4. Gatebriar should seek injunctive relief to prevent KBL from constructing a sports

bar/brew pub on Lot A on the basis that the Declaration prohibiting alcoholic

beverages from being made, sold or consumed on any parcels in the Plan is

enforceable as an equitable servitude.

Covenants affecting real property are mainly of two kinds. A restriction which is binding

only upon the covenantor and which is not intended to be a continuing charge on real estate is

classified as a personal covenant. Those restrictions which are so closely connected with real

estate that their benefits and burdens pass to subsequent purchasers and assignees of the real

estate are classified as covenants running with the land. DeSanno v. Earle, 117 A. 200, 202 (Pa.

1922).

By convention and as a historical fact, covenants running with the land evolved into two

types. This first type, which developed in the English common law courts, is known as a

covenant running at law or a real covenant. ROGER A. CUNNINGHAM ET AL, THE LAW OF

PROPERTY, 468 (West Publishing Co. 1984). The remedy sought by a person seeking to enforce

a real covenant is money damages. Id. at 484. The second type of covenant running with the

land, which developed in the English Chancery Court, is known as an equitable servitude. Id. at

468. The remedy sought by a person seeking to enforce an equitable servitude is equitable in

nature. Id. at 497. Pennsylvania law provides that an action in equity seeking injunctive relief is

available to enforce a covenant running with the land contained in a deed or recorded plan.

Peters v. Davis, 231 A.2d 748, 752 (Pa.1967); Doylestown Twp. v. Teeling, 635 A.2d 657, 660

(Pa. Cmwlth. 1993), appeal denied, 653 A.2d 1234 (Pa. 1994). Since Gatebriar wants to stop the

construction of the sports bar/brew pub on Lot A, it would want equitable relief in the form of an

injunction. Therefore, Gatebriar must prove that the restriction prohibiting alcoholic beverages

from being made, sold, or consumed on any parcels in the Plan is enforceable as an equitable

servitude.

To qualify as an equitable servitude, the restriction concerning the use of the land must

satisfy several requirements. First, the restriction must touch and concern the land. This

requirement generally involves the imposition of some benefit or burden upon the land by some

restriction of its use. Goldberg v. Nicola, 178 A. 809, 810 (Pa.1935); WILLIAM B. STOEBUCK &

DALE A. WHITMAN, THE LAW OF PROPERTY, § 8.15 (West Publishing Co. 3d ed. 2000). In this

case, the restriction in the Declaration governing the use of the lots within the Plan touches and

concerns the land because it imposes a benefit or burden by prohibiting the manufacture, sale, or

consumption of alcoholic beverages on any parcels in the Plan.

The second requirement in determining whether a restriction concerning the land is

enforceable as an equitable servitude is the intent that the restriction run with the land. DeSanno,

117 A. at 202. “[T]o ascertain such intent, resort may be had to the words of the covenant read

in the light of the surroundings of the parties and the subject of the grant.” Id. Neither formal

nor specific technical language is needed to create an equitable servitude. See, Logston v. Penndale, Inc., 576 A.2d 59, 62 (Pa. Super. 1990), citing Elec. City Land & Improvement Co. v. West Ridge Coal Co., 41 A. 458, 462 (Pa. 1898). Nevertheless, the use of certain terms and

language normally indicates that a covenant running with the land is intended. In this case, the

restriction on the use of the two parcels in the Plan’s Declaration states, “The benefits and

42

Page 47: JULY 2018 PENNSYLVANIA BAR EXAMINATION · JULY 2018 PENNSYLVANIA BAR EXAMINATION . Essay Questions and Examiners’ Analyses. and . Performance Test . Pennsylvania Board of Law Examiners

burdens of this restriction shall be applicable to all grantees, their heirs, successors, and assigns.”

Language stating that a particular restriction is binding upon the grantees’ heirs and assigns is

“generally decisive of the question” whether a covenant is intended to run with the land. Leh v. Burke, 331 A.2d 755, 760 (Pa. Super. 1974); RESTATEMENT (THIRD) OF PROPERTY

(SERVITUDES) § 2.2, cmt. d (2000).

The final requirement2 needed to enforce a covenant running with the land as an equitable

servitude is that the party against whom the covenant is to be enforced had notice of the

restriction. RALPH E. BOYER ET AL, THE LAW OF PROPERTY: AN INTRODUCTORY SURVEY, 323 (West

Publishing Co. 4th ed. 1991). In this case, KBL did not have actual notice of the Declaration

requiring that no alcoholic beverages be made, sold, or consumed on Lot A. Further, no

reference to the restriction was included in his deed to Lot A. KBL, however, cannot use either

the absence of actual notice of the restriction or the lack of any reference to the restriction in his

deed to avoid compliance with the restriction on the use of the property. Longstanding

Pennsylvania law provides not only that a property owner had the duty to become aware of the

recorded restrictions in his chain of title but also that the property owner would be bound by such

restrictions in the absence of actual notice. Finley v. Glenn, 154 A. 299, 301 (Pa. 1931); accord,

Doylestown Twp., 635 A.2d at 661; Leh, 331 A.2d at 761. Because the restriction here was in the

recorded Declaration, KBL will be deemed to have constructive notice of the restriction and the

restriction can be enforced against it.

In conclusion, Gatebriar should seek injunctive relief to prevent KBL from constructing

its sports bar/brew pub on Lot A because the restriction in the Declaration prohibiting alcoholic

beverages from being made, sold, or consumed on any parcels in the Plan is enforceable as a

common law equitable servitude.

2 Because an equitable servitude evolved under common law into an interest in land, some commentators include an

instrument which complies with the statute of frauds as another necessary element. RALPH E. BOYER ET AL, THE

LAW OF PROPERTY: AN INTRODUCTORY SURVEY, 325-26 (West Publishing Co. 4th ed. 1991). In this case, the

written and recorded Declaration would satisfy the writing requirement. Privity of estate also was a required

element under the common law for a covenant to run at law. “The traditional privity doctrine was grounded in the

notion that because covenants did not create property rights, they could run only if there was an ‘identification’

between the original covenantee or covenantor and the successor. Privity supplied that identification.”

RESTATEMENT (THIRD) OF PROPERTY, (SERVITUDES) 5 Intro. Note (2000). The traditional requirement of

privity has been discarded by the Restatement “because its conceptual basis has disappeared with the modern

recognition that covenants create property rights, it is not supported by any modern cases, and it produces unsound

results.” Id. Even if privity was a requirement under Pennsylvania law, a privity analysis would not be necessary in

this case because privity of estate is not required for a covenant to run in equity. BOYER, 323.

43

Page 48: JULY 2018 PENNSYLVANIA BAR EXAMINATION · JULY 2018 PENNSYLVANIA BAR EXAMINATION . Essay Questions and Examiners’ Analyses. and . Performance Test . Pennsylvania Board of Law Examiners

Question No. 5: Grading Guidelines

1. Contract Formation - Offer, Acceptance, and Counter-Offer

Comments: Candidates should define and discuss two of the basic elements necessary to form a

contract under the common law: offer and acceptance. Candidates should recognize that a reply

to an offer which changes the terms and conditions of the offer is not an acceptance, but a

counter-offer that normally terminates the power to accept the original offer. Candidates further

should recognize that an offeree’s manifestation of a contrary intent can overcome the normal

operation of a counter-offer as a rejection of the original offer and thereby allow the offeree to

accept the original offer.

5 Points

2. Concurrent Estates – Tenants in Common, Joint Tenancy with Right of

Survivorship and Tenancy by the Entireties

Comments: Candidates should discuss the differences between the concurrent estates of joint

tenancy with right of survivorship and tenancy in common and the present common law and

statutory presumption favoring tenancy in common. Candidates should analyze the particular

language in Brenda’s deed and reach a well-reasoned conclusion concerning whether the

presumption favoring tenancy in common was overcome in this case.

Candidates should discuss Pennsylvania law’s presumption that a conveyance to a husband and

wife creates a tenancy by the entireties even when the words of the instrument fail to state the

existence of the marriage. Candidates should recognize that an estate held by the entireties

cannot be attached or sold by a creditor of one of the tenants as long as both tenants are alive and

remain married. Candidates should apply these principles to the stated facts and reach the

conclusion that Big Bank’s attempt to force a sale of the entireties property recover on its

judgment against Al will not be successful.

7 Points

3. Damages for Breach of Contract –Mitigation of Damages

Comments: Candidates should recognize that a party injured by breach of contract is required to

mitigate damages and that a failure to reasonably attempt to mitigate damages reduces the

injured party’s recovery by the amount of avoidable losses. Based upon the stated facts,

candidates should apply the law in reaching a well-reasoned conclusion concerning whether Al

made reasonable efforts to reduce his damages by finding comparable employment.

3 Points

44

Page 49: JULY 2018 PENNSYLVANIA BAR EXAMINATION · JULY 2018 PENNSYLVANIA BAR EXAMINATION . Essay Questions and Examiners’ Analyses. and . Performance Test . Pennsylvania Board of Law Examiners

4. Enforcement of an Equitable Servitude

Comments: Candidates should recognize that the Declaration restricting the use of the lots in the

Plan is enforceable as an equitable servitude. Candidates shall identify the elements necessary

for an equitable servitude to benefit successive owners and to be binding upon successive owners

and apply the elements to the stated facts. Candidates should conclude their analysis by

discussing the specific relief to enforce the restriction.

5 Points

45

Page 50: JULY 2018 PENNSYLVANIA BAR EXAMINATION · JULY 2018 PENNSYLVANIA BAR EXAMINATION . Essay Questions and Examiners’ Analyses. and . Performance Test . Pennsylvania Board of Law Examiners

Question No. 6

On November 8, 2015, 25-year-old Penny was driving in C County, Pennsylvania, when her car

left the road during a storm and crashed. Her head hit the windshield. During Penny’s call to 911, she

stated, “This was all my fault. I shouldn’t have driven so fast on that slippery road.” When emergency

medical technicians (“EMTs”) arrived, Penny was not breathing. The EMTs resuscitated her and

transported her to the C County Hospital (“Hospital”).

At Hospital, an x-ray revealed a fracture with compression to the front of Penny’s skull. Dr. Stone

(“Stone”), a Hospital employee and an authorized member of its surgical staff, was on call. Stone was

new to Hospital, having just completed his residency. Hospital had hired Stone knowing he was very

inexperienced. After reviewing Penny’s x-rays and vital signs, Stone immediately performed emergency

surgery to stop the bleeding in Penny’s brain and relieve pressure in her skull. Penny was Stone’s first

unsupervised brain surgery. Hospital did not have any protocols in place requiring inexperienced surgeons

to consult with another surgeon before performing brain surgery. After repairing all visible sources of

bleeding, Stone noted a small amount of blood still seeping, but could not immediately see its source. He

closed without searching further for the origin of the bleeding, hoping it would stop on its own.

After surgery, Penny was placed in a medically induced coma and given medications to minimize

swelling in her brain; however, her condition worsened. Stone monitored Penny’s condition for 24 hours

before concluding that there was still some bleeding in Penny’s brain. In a second surgery the day after

the first, Stone found and repaired a small bleed in an artery. Penny then improved. On November 16,

2015, Penny was brought out of the coma. She was suffering significant, permanent memory loss. That

same day, Stone fully discussed with Penny and her parents all his actions and decisions in treating her,

and they took notes of the discussion.

46

Page 51: JULY 2018 PENNSYLVANIA BAR EXAMINATION · JULY 2018 PENNSYLVANIA BAR EXAMINATION . Essay Questions and Examiners’ Analyses. and . Performance Test . Pennsylvania Board of Law Examiners

Based on a medical opinion Penny’s parents obtained from another doctor on November 20, 2015,

Penny filed a civil complaint against Stone and Hospital in the C County Court of Common Pleas. Penny

alleged that Stone acted negligently and deviated from acceptable physician standards, either by missing a

damaged artery or by nicking a previously undamaged artery during the first surgery. Based on the

medical opinion, Penny claimed her memory loss resulted from the continued bleeding in her brain after

the first surgery.

1. Penny asserted negligence claims against Hospital based on both direct and vicarious

liability relating to Stone’s acts or omissions. Assuming Penny’s claim was properly filed,

how would the court analyze each of these two claims?

2. Assume, for this question only, that Penny filed her negligence lawsuit on November 30,

2017. If the defense properly seeks dismissal of Penny’s claim asserting that the claim was

untimely, how would the court likely rule?

3. Assume, for this question only, that after Penny files her lawsuit, Hospital implements a

new protocol requiring inexperienced surgeons to consult with an experienced surgeon

before performing brain surgery. Penny wants to introduce evidence of the new protocol at

trial. Assume this evidence is otherwise relevant and not subject to hearsay objections.

Under Pennsylvania rules of evidence, if Penny offers this evidence to demonstrate that the

former absence of such protocols constituted negligence by Hospital, how should the court

rule on admissibility of the evidence for that purpose?

4. Assume Penny’s statement to the 911 operator will be admissible at trial regarding Penny’s

negligence claims against Hospital and Stone. Assume the jury, having been properly

instructed, finds that Penny was negligent, along with Stone and Hospital, and all three

were at fault in causing her injury. Under Pennsylvania law:

(a) If the jury finds Stone and Hospital each 30% at fault and finds Penny 40% at fault for

her injuries, how does this affect the amount, if any, of the verdict Penny may recover?

(b) If the jury finds Stone and Hospital each 20% at fault and Penny 60% at fault for her

injuries, how does this affect the amount, if any, of the verdict Penny may recover?

47

Page 52: JULY 2018 PENNSYLVANIA BAR EXAMINATION · JULY 2018 PENNSYLVANIA BAR EXAMINATION . Essay Questions and Examiners’ Analyses. and . Performance Test . Pennsylvania Board of Law Examiners

Question No. 6: Examiner’s Analysis

1. Penny may assert a claim of direct liability for negligent hiring and supervision, as well as a

claim of vicarious liability against Hospital.

Direct negligence

“Pennsylvania recognizes the doctrine of corporate negligence as a basis for hospital liability

separate from the liability of the practitioners who actually have rendered medical care to patients.” Rauch v. Mike-Mayer, 783 A.2d 815, 826 (Pa. Super. 2001). As with all negligence claims, a claim of negligence

against a hospital requires proof of all four elements of that claim: a duty of care, a breach of that duty,

resulting injury, and damages. Id. at 826-27. In Pennsylvania, a hospital’s specific duty of care to

emergency patients is governed by the Restatement (Second) of Torts § 323 (1965):

One who undertakes, gratuitously or for consideration, to render services to another which

he should recognize as necessary for the protection of the other’s person or things, is

subject to liability to the other for physical harm resulting from his failure to exercise

reasonable care to perform his undertaking, if

(a) his failure to exercise such care increases the risk of such harm, or

(b) the harm is suffered because of the other’s reliance upon the undertaking.

Thompson v. Nason Hosp., 527 Pa. 330, 340, 591 A.2d 703, 708 (1991) (quoting Riddle Mem’l Hosp. v.

Dohan, 504 Pa. 571, 576-77, 475 A.2d 1314, 1316 (1984) (quoting Restatement (Second) of Torts § 323

(1965))).

A hospital owes a non-delegable duty to ensure the patient’s safety and wellbeing while at the

hospital. “Therefore, an injured party does not have to rely on and establish the negligence of a third

party” in order to bring a direct claim against a hospital. Thompson, 527 Pa. at 339, 591 A.2d at 707.

Pertinent here, the Hospital’s duty to its patients includes “a duty to select and retain only competent

physicians . . . [and] a duty to [supervise] all persons who practice medicine within its walls[,]” as well as

“a duty to formulate, adopt and enforce adequate rules and policies to ensure quality care for the patients.”

Id. at 339-40, 591 A.2d at 707.

For a hospital to be liable to a patient for direct negligence, the plaintiff must “show that the

hospital had actual or constructive knowledge of the defect or procedures [that] created the harm[, and] the

hospital’s negligence must have been a substantial factor in bringing about the harm to the injured party.”

Id. at 341, 591 A.2d at 708. Thus, the plaintiff must show that the hospital’s negligence was a factual

cause in bringing about her injury. Gorman v. Costello, 929 A.2d 1208, 1212-13 (Pa. Super. 2007)

(quoting SSJI Civ 3.15).

[F]actual cause does not mean it is the only, primary, or even the most important factor in

causing the injury. A cause may be found to be a factual cause as long as it contributes to

the injury in a way that is not minimal or insignificant. To be a contributing factor, the

48

Page 53: JULY 2018 PENNSYLVANIA BAR EXAMINATION · JULY 2018 PENNSYLVANIA BAR EXAMINATION . Essay Questions and Examiners’ Analyses. and . Performance Test . Pennsylvania Board of Law Examiners

defendant’s conduct need not be the only factor. The fact that some other cause concurs

with the negligence of the defendant in producing an injury does not relieve the defendant

from liability as long as [his] [her] own negligence is a factual cause of the injury.

Id. at 1212 (internal quotations and citation omitted).

Applying this rule to the facts of the question, Hospital owed a duty of care to Penny to retain

competent surgeons, to oversee their activities, and to have proper protocols in place. To prove direct

liability against Hospital, Penny must show that Hospital breached its duty of care because Stone was not

competent, because it did not adequately supervise his surgical activities, or because it did not implement

protocols sufficient to assure adequate supervision or timely follow-up care after a patient’s condition

worsens post-surgery. Relevant to these issues is the fact that Hospital knew of Stone’s level of

inexperience, but did not put protocols in place that might have provided mandatory consultation and

supervision in connection with his performance of emergency brain surgery. The finder of fact would

consider these issues in deciding whether Hospital violated its duty of care.

Penny must also prove that Hospital’s conduct was a factual cause of her memory loss. However,

to plead a claim of direct liability against Hospital, Penny does not need to allege that Stone was negligent.

Vicarious liability

As a general rule, a master has the right to exercise control over the physical activities of the

servant within the time of service and is vicariously liable for the servant’s negligent acts committed

within the scope of his employment. Smalich v. Westfall, 440 Pa. 409, 415, 269 A.2d 476, 481 (1970)

(quoting Restatement (Second) of Agency § 219 (1958) (further citation omitted)). More specifically, a

hospital owes a duty to its patients and may be liable under agency theory, as the principal, for the

negligence of surgical personnel. Tonsic v. Wagner, 458 Pa. 246, 253, 329 A.2d 497, 501 (1974). See also 40 P.S. § 1303.516 (A hospital may be vicariously liable for acts of a health care provider if a

reasonable person would justifiably believe the care in question was being rendered by the hospital or its

agents, or if the care was represented as being rendered by the hospital or its agents.).

To sustain a claim of vicarious liability against Hospital, Penny must satisfy the elements of a

negligence claim against Stone: a duty owed by Stone to Penny, a breach of that duty, and damages

directly resulting from the breach. Here, the facts do not indicate that Stone made any attempt to consult

any other surgeon or doctor, or to seek assistance with Penny’s surgery. Further, he concluded the surgery

and closed despite apparent ongoing bleeding in Penny’s brain.

As the facts indicate no dispute that Stone was an employee on duty in Hospital and was authorized

to perform surgery, a viable negligence claim against him will also constitute a viable claim of vicarious

liability against Hospital.

2. The court would likely rule that Penny’s complaint was filed outside the two-year statute of

limitations for a negligence claim and dismiss it.

The Pennsylvania Judicial Code (“Code”) (42 Pa.C.S. §§ 5501-5574) sets forth the time periods

within which an action must be filed. The Code specifically provides that a tort action must be

49

Page 54: JULY 2018 PENNSYLVANIA BAR EXAMINATION · JULY 2018 PENNSYLVANIA BAR EXAMINATION . Essay Questions and Examiners’ Analyses. and . Performance Test . Pennsylvania Board of Law Examiners

commenced within two years from the date the cause of action accrues. 42 Pa.C.S. § 5524 (2017). “Once

the prescribed statutory period has expired, the party is barred from bringing suit unless it is established

that an exception to the general rule applies which acts to toll the running of the statute.” Bickford v. Johnson, 368 Pa. Super. 211, 220, 533 A.2d 1029, 1033 (1987). The running of the statute of limitations

in a tort case may be tolled during a period when, despite the exercise of reasonable diligence, the plaintiff

was unable to discover her injury or its cause. Id., 368 Pa. Super. at 215, 533 A.2d at 1031. In such an

instance, the cause of action accrues when the plaintiff reasonably should be aware of the injury and the

facts supporting a potential tort claim. See id.

Applying this statute to Penny’s claim, she was specifically made aware of all the events giving

rise to her claim when she was brought out of her coma, i.e., on November 16, 2015, and her cause of

action accrued at that time. Her memory loss would have been readily apparent. Further, Stone’s

discussion with her on that date explained all the facts concerning her surgeries, thus eliminating any

discovery rule issue. See id. Even if it can be argued that Penny did not know the cause of her injury on

November 16, 2015, at the latest she was aware on November, 20, 2015, when her parents got the second

opinion. Thus, the statute of limitations therefore ran on Penny’s claim, at the latest, on November 20,

2017. She filed her action on November 30, 2017, which means that her claim was filed outside the statute

of limitations. Consequently, under the facts assumed for this portion of the question, Penny’s claim will

be dismissed.

3. The court should rule that evidence of subsequent remedial measures is inadmissible to

suggest prior negligence.

The implementation of new protocols after Penny files her lawsuit, requiring that inexperienced

surgeons consult with an experienced surgeon before performing brain surgery, would be classified as a

subsequent remedial measure. See Pa.R.E. 407.

When measures are taken by a party that would have made an earlier injury or harm less

likely to occur, evidence of the subsequent measures is not admissible against that party to

prove:

negligence;

culpable conduct;

a defect in a product or its design; or

a need for a warning or instruction.

But the court may admit this evidence for another purpose such as impeachment or – if

disputed – proving ownership, control, or the feasibility of precautionary measures.

Pa.R.E. 407 (2013). Specifically, Pennsylvania law precludes admission of any evidence of the review

process or the resulting change in protocols, when offered for the purpose of imputing antecedent

negligence. See Larch v. Haverford State Hosp., 152 Pa. Cmwlth. 459, 465, 620 A.2d 37, 40-41 (1993).

The rationale behind this rule is twofold. First, because a negligence action depends on the

standard of care applicable at the time of the relevant events, subsequent repairs are not relevant; they may

have arisen from new or different circumstances, or from a change in the applicable duty of care. Second,

50

Page 55: JULY 2018 PENNSYLVANIA BAR EXAMINATION · JULY 2018 PENNSYLVANIA BAR EXAMINATION . Essay Questions and Examiners’ Analyses. and . Performance Test . Pennsylvania Board of Law Examiners

social policy encourages remedial measures that will enhance safety, and admitting evidence of remedial

measures against the party implementing them might improperly discourage that party from undertaking

such measures. Duchess v. Langston Corp., 564 Pa. 529, 535-36, 769 A.2d 1131, 1134 (2001).

Under Rule 407, however, if feasibility is disputed and the trial court deems the evidence relevant

to the question of whether the protocols were feasible for implementation prior to Penny’s surgery, the

evidence will be admissible for that purpose. Feasibility is generally determined based on considerations

such as cost, practicality, and technological possibility. Duchess, 564 Pa. at 554, 769 A.2d at 1146.

Duchess cautions that the exception for feasibility should be narrowly applied so that the exception does

not swallow the rule. Id.

Applying the rule to the facts of the question, the facts as given do not suggest any of the

exceptions listed in Rule 407. The implementation of new protocols would not be admissible merely for

the purpose of imputing antecedent negligence to the defendants.

4.a. If Penny is found contributorily negligent, she can still recover if she was less than 50%

responsible for her injury.

Contributory/comparative negligence is an affirmative defense that defendants can assert to lower

or eliminate the defendant’s liability. See Pa.R.C.P. No. 1030; 42 Pa. C.S. § 7102. If a plaintiff makes out

a prima facie case of negligence, the defendant has the burden of proof to show that the plaintiff was

contributorily negligent. Contributory negligence is shown by “disproving the use of due care on the part

of plaintiff by a preponderance of the evidence.” Smith v. Port Auth. Transit, 257 Pa. Super 66, 70-71, 390

A.2d 249, 251 (1978).

Pennsylvania’s comparative negligence statute provides, in pertinent part:

§ 7102. Comparative negligence.

(a) General rule. - - In all actions brought to recover damages for negligence resulting in death or

injury to person or property, the fact that the plaintiff may have been guilty of contributory

negligence shall not bar a recovery by the plaintiff or his legal representative where such

negligence was not greater than the causal negligence of the defendant or defendants against whom

recovery is sought, but any damages sustained by the plaintiff shall be diminished in proportion to

the amount of negligence attributed to the plaintiff.

42 Pa. C.S. § 7102(a) (2011).

Contributory negligence is neglect of the duty imposed upon a person to exercise ordinary care for

his or her own protection and safety that is a legally contributing cause of an injury. Trayer v. King, 241 Pa. Super. 86, 90, 359 A.2d 800, 802 (1976). The contributory negligence of a plaintiff, like the

negligence of a defendant, is lack of due care under the circumstances. Argo v. Goodstein, 438 Pa. 468,

480, 265 A.2d 783, 789 (1970). A plaintiff is contributorily negligent with respect to injuries which are

received as a result of a failure on her part to observe and avoid an obvious condition that ordinary care for

her safety would have disclosed. Skalos v. Higgins, 303 Pa. Super. 107, 114, 449 A.2d 601, 604 (1982).

51

Page 56: JULY 2018 PENNSYLVANIA BAR EXAMINATION · JULY 2018 PENNSYLVANIA BAR EXAMINATION . Essay Questions and Examiners’ Analyses. and . Performance Test . Pennsylvania Board of Law Examiners

Penny’s acknowledgment that she was driving too fast for the road conditions and was at fault for

the accident could support a defense of contributory negligence. Fact issues would remain for trial

concerning the cause or causes of Penny’s memory loss.

Case law has established that the party causing an injury and a physician subsequently aggravating

that injury or causing a new one are not joint tortfeasors because they act independently. See, e.g., Smith v. Pulcinella, 440 Pa. Super. 525, 530, 656 A.2d 494, 497 (1995). However, that does not preclude

consideration of the parties’ relative fault under principles of comparative negligence. As one court

explained:

The critical issue . . . with respect to the comparative negligence and contributory

negligence questions is whether the negligence of various defendants and plaintiff were

substantial factors in causing the injuries. Where an automobile accident occurs, and an

injured party is then treated negligently by medical care personnel, one party might be

found causally negligent with respect to the accident and the medical personnel also be

found causally negligent in treatment of the injuries. The cause of action with respect to

both defendants might be considered in the same litigation and the jury would apportion

the negligence of the two tortfeasors according to the effect each had on the ultimate

injuries. Similarly, a defendant might negligently cause a “fender bender” accident

which results in injuries which are unexpectedly severe because the other driver was not

using a seat belt. The question in either case should be whether the negligence of the

given party was a substantial factor in causing the injury sustained by plaintiff; not

whether the negligence was a substantial factor in causing the vehicular impact.

Turner v. Scaife, 44 D. & C.3d 349, 354-55 (C.P. Lycoming 1987).

Here, the facts assume that a jury finds Penny partly at fault for her injury. Thus, the facts likewise

assume that Penny’s conduct and the defendants’ conduct joined in causing that injury.

“Pennsylvania[’s] Comparative Negligence Act replaced the harsh common law doctrine of

‘contributory negligence’ under which a plaintiff whose own negligence, however slight, contributed to

the happening of the accident in a proximate way, was barred from recovery.” Elder v. Orluck, 511 Pa.

402, 416, 515 A.2d 517, 524 (1986). Under the Act, a plaintiff can recover provided her fault is 50% or

less of the cause of her harm. The Elder court held that “[c]omparison of the plaintiff’s negligence to that

of the combined negligence of all defendants is consistent with and furthers the intent of the Act. It

insures that an injured plaintiff, who is otherwise entitled to a recovery, will not go uncompensated

because of the number of defendants who contributed to [her] injuries.” Id.

Applying the statute to the facts here, Penny was 40% at fault for her injury, while the combined

fault of the defendant’s was 60%. Accordingly, Penny will recover under this scenario, but her recovery

will be reduced by the percentage of her fault.

4.b. If the jury finds Penny was 60% responsible for her injury, she will not be able to recover for

her injury under the Comparative Negligence Act.

52

Page 57: JULY 2018 PENNSYLVANIA BAR EXAMINATION · JULY 2018 PENNSYLVANIA BAR EXAMINATION . Essay Questions and Examiners’ Analyses. and . Performance Test . Pennsylvania Board of Law Examiners

When a plaintiff’s negligence contributes more than 50% to her injury, the Act is a bar to her

recovery. 42 Pa. C.S. § 7102(a); See also Elder, 511 Pa. at 416, 515 A.2d at 524.

Under this scenario, Penny was 60% at fault for her injury, while the sum of the defendants’

percentage of fault was 40%. Thus, under the Comparative Negligence Act, Penny will recover nothing.

53

Page 58: JULY 2018 PENNSYLVANIA BAR EXAMINATION · JULY 2018 PENNSYLVANIA BAR EXAMINATION . Essay Questions and Examiners’ Analyses. and . Performance Test . Pennsylvania Board of Law Examiners

Question No. 6: Grading Guidelines

1. Torts – Professional Negligence and Vicarious Liability

Comments: The applicant should recognize that two claims are possible against the hospital – (1) direct

liability for negligent hiring and supervision, and (2) vicarious liability. The applicant should set forth the

elements of each cause of action and apply them to the facts of the question.

8 Points

2. Civil Procedure – Statute of Limitations

Comments: The applicant should recognize the statute of limitations issue and apply the two year statute

to the facts in the question and conclude that Penny’s complaint falls outside the statute.

3 Points

3. Evidence – Subsequent remedial measures

Comments: The applicant should recognize that Pennsylvania Rules of Evidence preclude admission of

evidence concerning subsequent remedial measures for the purpose of proving antecedent negligence, but

that the evidence may be admissible for another purpose, such as to establish feasibility.

4 points

4. Affirmative Defense – Comparative Negligence

Comments: The applicant should recognize that Pennsylvania’s comparative negligence statute governs

this affirmative defense and apply the defense to the facts in the relevant facts.

5 Points

54

Page 59: JULY 2018 PENNSYLVANIA BAR EXAMINATION · JULY 2018 PENNSYLVANIA BAR EXAMINATION . Essay Questions and Examiners’ Analyses. and . Performance Test . Pennsylvania Board of Law Examiners

PT

Supreme Court of Pennsylvania

Pennsylvania Board of Law Examiners

Pennsylvania Bar Examination

February 24 and 25, 2015

PERFORMANCE TEST

February 24, 2015

Use GRAY covered book for your answer to the Performance Test.

© 2015 Pennsylvania Board of Law Examiners

PLACE BAR CODED APPLICANT LABEL HERE

Question Number 3

on Examplify

55

Page 60: JULY 2018 PENNSYLVANIA BAR EXAMINATION · JULY 2018 PENNSYLVANIA BAR EXAMINATION . Essay Questions and Examiners’ Analyses. and . Performance Test . Pennsylvania Board of Law Examiners

Table of Contents

FILE

1. Assignment Memorandum ................................................................................................... 1

2. Plaintiff’s Complaint ............................................................................................................ 2

3. Deed to Plaintiff’s Property ................................................................................................. 5

4. Deed to Defendant’s Property .............................................................................................. 6

5. Map of Parcels in Question .................................................................................................. 7

6. Formatting Memorandum for Drafting Preliminary Objections .......................................... 8

7. Sample Preliminary Objections ........................................................................................... 9

LIBRARY

1. Wolfe v. Porter, 592 A.2d 716 (Pa.Super. 1991) ................................................................. 10

2. Zeglin v. Gahagen, 812 A.2d 558 (Pa. 2002) ...................................................................... 13

56

Page 61: JULY 2018 PENNSYLVANIA BAR EXAMINATION · JULY 2018 PENNSYLVANIA BAR EXAMINATION . Essay Questions and Examiners’ Analyses. and . Performance Test . Pennsylvania Board of Law Examiners

FILE

57

Page 62: JULY 2018 PENNSYLVANIA BAR EXAMINATION · JULY 2018 PENNSYLVANIA BAR EXAMINATION . Essay Questions and Examiners’ Analyses. and . Performance Test . Pennsylvania Board of Law Examiners

Memorandum

TO: Applicants

FROM: Mark Kerwick, Managing Partner

RE: Assignment to Draft Preliminary Objections

DATE: February 27, 2018

Ten (10) days ago, our client, George Gilbert, was properly served with a complaint concerning the ownership

of a parcel of land near Lake Anne in Lake Anne Township, Luke County, Pennsylvania. The complaint was

filed in the Court of Common Pleas of Luke County, Pennsylvania. The plaintiff, Paul Kerrigan, claims

ownership of a tract of land measuring forty feet (40’) by fifty feet (50’), even though record title to the

property is in the name of George Gilbert.

The first part of your assignment is to draft two preliminary objections to the complaint asserting that the

complaint is legally insufficient (demurrer). The first preliminary objection in the nature of a demurrer is based

on Plaintiff’s failure to meet the elements of adverse possession. The second preliminary objection in the nature

of a demurrer is based on plaintiff’s failure to meet the elements of consentable/aquiescence in a boundary.

Although there are other preliminary objections that we could raise, your assignment is limited to the preceding

preliminary objections as I will be drafting any additional objections. Do not raise tacking or privity as part of

your preliminary objection based upon plaintiff’s failure to meet the elements of consentable/aquiescence in a

boundary. The second part of your assignment is to draft only the legal argument section of a brief in support of

the two preliminary objections. I will draft the remaining sections of the brief. Your legal argument section

should integrate relevant facts and set forth the law that will convince the court to grant the two preliminary

objections. Place the legal argument portion of the assignment at the end of your preliminary objections and

separate your legal argument with the heading “Legal Argument.”

Included in the attached File is the plaintiff’s complaint (attached to which are the deeds to the parties’

properties, and a map of the parcels in question), a formatting memorandum on drafting preliminary objections,

and a sample form of preliminary objection. Included in the attached Library is caselaw relevant to the two

objections. You should only use facts contained in the File, and you should only use the attached cases for your

preliminary objections and legal argument section. Do not rely upon your personal knowledge of these issues

or on cases and statutes not included in the Library. Instead you should base your objections, analysis, and

conclusions only upon the documents provided in the File and the Library.

Please set forth each preliminary objection and your legal argument under separate headings. In your legal

argument section, provide the applicable standards for granting preliminary objections and a reasoned legal

analysis supporting each preliminary objection. When citing to any authority, Bluebook citations are not

necessary; however, you must include sufficient informal citations to the appropriate authority, such that I will

know to which document you are referring.

58

Page 63: JULY 2018 PENNSYLVANIA BAR EXAMINATION · JULY 2018 PENNSYLVANIA BAR EXAMINATION . Essay Questions and Examiners’ Analyses. and . Performance Test . Pennsylvania Board of Law Examiners

PAUL KERRIGAN, Plaintiff : In the Court of Common Pleas of Luke County

:

v. : Civil Action - Law

: Action to Quiet Title

GEORGE GILBERT, Defendant : No. 2017 CIV 6241970

COMPLAINT

AND NOW, comes the Plaintiff, Paul Kerrigan, and in support of his Complaint avers as follows:

1. Plaintiff, Paul Kerrigan, is an adult competent individual who resides at Lot 3, Lake Anne

Township, Luke County, Pennsylvania.

2. Defendant, George Gilbert, is an adult competent individual who resides at 12 Church Street,

Lake Anne Township, Luke County, Pennsylvania.

3. Plaintiff is the titled owner of Lot 3 in the Lake Anne Subdivision, said property being more

particularly described in the Deed recorded with the Luke County, Pennsylvania, Recorder of Deeds Office at

instrument number 06-03-19952, and which is attached hereto as “Exhibit A.”

4. Lot 3 is a lakefront parcel measuring 40’ x 100’ that runs from the Lake on the front side of the

property to the public roadway on the back side of the property, with the front and back sides measuring 40’

each, and the sides of the property running from the lake to the road being 100’ each.

5. Defendant is the titled owner of Lot 6 in the Lake Anne Subdivision, said property being more

particularly described in the Deed recorded with the Luke County, Pennsylvania, Recorder of Deeds Office at

instrument number 09-04-20041, and which is attached hereto as “Exhibit B.”

6. Lot 6 is a large open field, consisting of approximately 50+ acres, with no fencing, trees or other

distinguishing characteristics.

7. Plaintiff and his predecessor in title have been the titled owners to Lot 3 for in excess of fifty-

three (53) years, with Plaintiff having purchased the parcel ten (10) years ago.

8. For no fewer than thirty (30) years, Plaintiff and his predecessor in title have used an area of land

40’ wide and 50’deep, which is located directly across the public road from the back side of Lot 3 and which is

encompassed as part of Lot 6. Plaintiff and his predecessor in title have been in actual, open, hostile,

59

Page 64: JULY 2018 PENNSYLVANIA BAR EXAMINATION · JULY 2018 PENNSYLVANIA BAR EXAMINATION . Essay Questions and Examiners’ Analyses. and . Performance Test . Pennsylvania Board of Law Examiners

continuous and exclusive possession of the 40’ x 50’ tract contained within Lot 6 for at least thirty (30) years.

A map of the Lake Anne Subdivision is attached hereto as “Exhibit C,” and Plaintiff’s counsel has drawn a

dotted line thereon depicting the 40’x50’ tract in question.

COUNT I – ADVERSE POSSESSION

9. Plaintiff incorporates each and every paragraph by reference as if more fully set forth herein.

10. Plaintiff and his predecessor in title have adversely possessed the 40’ x 50’ tract of land

referenced above for a period of time in excess of twenty-one (21) years.

11. Plaintiff and his predecessor in title have occupied and utilized said tract as their own, for their

own exclusive use, including parking of vehicles and conducting recreational activities. Plaintiff and his

predecessors have continuously exerted actual, open, hostile and exclusive possession of said tract for more

than twenty-one (21) years.

Wherefore, Plaintiff respectfully requests that the Court enter judgment in his favor and against

Defendant, declaring and adjudging that Plaintiff owns absolutely and is entitled to quiet and peaceful

possession of the 40’ x 50’ tract encompassed by the Deed to Lot 6, and which is located directly across the

public dirt roadway from Lot 3, and that the Recorder of Deeds be Ordered to execute a deed that can be

properly recorded to reflect that Plaintiff is the sole owner of the 40’ x 50’ tract of land.

COUNT II – CONSENTABLE/ACQUIESCENCE IN A BOUNDARY

12. Plaintiff incorporates each and every paragraph by reference as if more fully set forth herein.

13. Plaintiff and his predecessor in title have possessed the 40’ x 50’ tract referenced above for more

than thirty (30) years up through and including the present. Defendant has acquiesced to Plaintiff’s ownership

of the tract.

Wherefore, Plaintiff respectfully requests that the Court enter judgment in his favor and against

Defendant, declaring and adjudging that Plaintiff owns absolutely and is entitled to quiet and peaceful

possession of the 40’ x 50’ tract encompassed by the Deed to Lot 6, and which is located directly across the

60

Page 65: JULY 2018 PENNSYLVANIA BAR EXAMINATION · JULY 2018 PENNSYLVANIA BAR EXAMINATION . Essay Questions and Examiners’ Analyses. and . Performance Test . Pennsylvania Board of Law Examiners

public dirt roadway from Lot 3, and that the Recorder of Deeds be Ordered to execute a deed that can be

properly recorded to reflect that Plaintiff is the sole owner of the 40’ x 50’ tract of land.

Respectfully Submitted,

/s/ Atticus Mason Atticus Mason, Esquire

1 Courthouse Square

Maintown, PA 99909

Counsel to Plaintiff

61

Page 66: JULY 2018 PENNSYLVANIA BAR EXAMINATION · JULY 2018 PENNSYLVANIA BAR EXAMINATION . Essay Questions and Examiners’ Analyses. and . Performance Test . Pennsylvania Board of Law Examiners

EXHIBIT A

Instrument No. 06-03-19952

THIS DEED

MADE THIS 1st day of February 2008,

Between Ethan Bishop, of the Town of Maintown, County of Luke, Commonwealth of Pennsylvania

(hereinafter “Grantor”), and

Paul Kerrigan, of the Town of Maintown, County of Luke, Commonwealth of Pennsylvania (hereinafter

“Grantee”),

WITNESSETH, that in consideration of One Hundred Fifty Thousand Dollars ($150,000), in hand paid,

the receipt whereof is hereby acknowledged; Grantor does hereby grant to Grantee, his Heirs and Assigns,

ALL that certain lot, piece or parcel of land situate, lying and being in the Township of Lake Anne,

County of Luke, and Commonwealth of Pennsylvania, bounded and described as follows, to wit:

All that certain piece or lot of land called and known as Lake Anne Subdivision Lot 3, in the Township

of Lake Anne, bounded and described as follows:

BEGINNING at a pin at which the high water mark of Lake Anne, Lot 3 and Lot 4 come together;

thence 100’ North to a pin where the public dirt road, Lot 3 and Lot 4 come together; thence 40’ West to a pin

where the public dirt road, Lot 3 and Lot 2 come together; thence 100’ South to a pin where the high water

mark of Lake Ann, Lot 2 and Lot 3 come together; thence 40’ East to the pin where the high water mark of

Lake Anne, Lot 3 and Lot 4 come together. Containing approximately four thousand (4,000) square feet.

BEING the same premise conveyed by Benjamin Maier to Ethan Bishop by Deed dated January 3, 1965,

and recorded in Luke County Deed Book 551, Page 400.

AND the said Grantor shall Warrant Generally the property hereby conveyed.

IN WITNESS WHEREOF, the Grantor has hereunto set his hand and seal the day and year first written

above.

WITNESS: /s/ Maria VonBergen GRANTOR: /s/Ethan Bishop

62

Page 67: JULY 2018 PENNSYLVANIA BAR EXAMINATION · JULY 2018 PENNSYLVANIA BAR EXAMINATION . Essay Questions and Examiners’ Analyses. and . Performance Test . Pennsylvania Board of Law Examiners

EXHIBIT B

Instrument No. 09-04-20041

THIS DEED

MADE THIS 1st day of February 2008,

Between Lilly Townsley, of the Town of Maintown, County of Luke, Commonwealth of Pennsylvania

(hereinafter “Grantor”), and

George Gilbert, of the Town of Maintown, County of Luke, Commonwealth of Pennsylvania

(hereinafter “Grantee”),

WITNESSETH, that in consideration of Two Hundred Thousand Dollars ($200,000), in hand paid, the

receipt whereof is hereby acknowledged; Grantor does hereby grant to Grantee, his Heirs and Assigns,

ALL that certain lot, piece or parcel of land situate, lying and being in the Township of Lake Anne,

County of Luke, and Commonwealth of Pennsylvania, bounded and described as follows, to wit:

All that certain piece or lot of land called and known as Lake Anne Subdivision Lot 6, in the Township

of Lake Anne, bounded and described as follows:

BEGINNING at a pin at which the public dirt roadway, Lot 6 and Lot 9 come together; thence 1500’

North to a pin where the Lot 6, Lot 8, and Lot 9 come together; thence 1500’ West to a pin where Lot 6, Lot 8,

and Lot 7 come together; thence 1500’ South to a pin where the public dirt road, Lot 6 and Lot 7 come together;

thence 1500’ East to a pin where the public dirt roadway, Lot 6 and Lot 9 come together. Containing

approximately two million two hundred fifty thousand (2,250,000) square feet.

BEING the same premise conveyed by Dorothy Fendick to Lilly Townsley by Deed dated August 4,

1961, and recorded in Luke County Deed Book 301, Page 895.

AND the said Grantor shall Warrant Generally the property hereby conveyed.

IN WITNESS WHEREOF, the Grantor has hereunto set his hand and seal the day and year first written

above.

WITNESS: /s/ Gretchen Arcaro GRANTOR: /s/ Lilly Townsley

63

Page 68: JULY 2018 PENNSYLVANIA BAR EXAMINATION · JULY 2018 PENNSYLVANIA BAR EXAMINATION . Essay Questions and Examiners’ Analyses. and . Performance Test . Pennsylvania Board of Law Examiners

EXHIBIT C

Map of Lake Anne Subdivision (not to scale)

Lot 8

1500’

Lot 6 Lot 7 Lot 9

Lake Anne

1500’

40’

Tract

In

Question 50’

40’

100’

Dirt Road

Lot 1 Lot 2 Lot 3 Lot 4 Lot 5

64

Page 69: JULY 2018 PENNSYLVANIA BAR EXAMINATION · JULY 2018 PENNSYLVANIA BAR EXAMINATION . Essay Questions and Examiners’ Analyses. and . Performance Test . Pennsylvania Board of Law Examiners

Big Law Firm

TO: All Associates and Law Clerks

FROM: Mark Kerwick, Managing Partner

RE: Formatting Memorandum for Drafting Preliminary Objections

DATE: July 1, 2018

Use the following guidelines for drafting preliminary objections:

1. A preliminary objection is a pleading that sets forth a challenge to the sufficiency of a pleading,

often a complaint, based on the contents of that pleading. Preliminary objections to a complaint are

filed before a defendant answers the complaint, and are filed in an attempt to have the complaint, or

a portion thereof, dismissed before proceeding with further litigation.

2. Two or more preliminary objections may be raised in one pleading.

3. The preliminary objections shall be divided into paragraphs numbered consecutively.

4. Each paragraph in the preliminary objections shall contain as far as practicable only one material

allegation.

5. A material allegation is an assertion of fact that is essential to the objection.

6. The material facts on which a cause of action or defense is based shall be stated in a concise and

summary form.

7. Any part of a pleading may be incorporated by reference in another part of the same pleading or in

another pleading in the same action.

8. A party may incorporate by reference any matter which is recorded or transcribed verbatim in the

office of the prothonotary, clerk of any court of record, recorder of deeds or register of wills of such

county.

9. More than one preliminary objection may be raised in the same pleading; preliminary objections are

limited to following the grounds, as enumerated in Pennsylvania Rule of Civil Procedure No. 1028:

(1) lack of jurisdiction over the subject matter of the action or the person of the defendant,

improper venue or improper form or service of a writ of summons or a complaint;

(2) failure of a pleading to conform to law or rule of court or inclusion of scandalous or

impertinent matter;

(3) insufficient specificity in a pleading;

(4) legal insufficiency of a pleading (demurrer);

10. Each preliminary objection must be stated in a separate section of the pleading, and must have its

own heading (see sample preliminary objection attached hereto).

11. A paralegal will place a caption containing all relevant information at the top of your preliminary

objections. The paralegal will also place an appropriate notice to plead, verification, proposed order,

and signature block at the end of the preliminary objections. Thus, do not include any of these items

in the preliminary objections.

12. A sample preliminary objection to a complaint (also with no caption, notice to plead, verification, or

signature block) is attached hereto for your reference.

65

Page 70: JULY 2018 PENNSYLVANIA BAR EXAMINATION · JULY 2018 PENNSYLVANIA BAR EXAMINATION . Essay Questions and Examiners’ Analyses. and . Performance Test . Pennsylvania Board of Law Examiners

Sample Preliminary Objection

1. Plaintiff filed its complaint in the Court of Common Pleas of Luke County, Pennsylvania,

alleging that Plaintiff performed professional engineering services on Defendant’s behalf, and that Defendant

failed to pay Plaintiff for those services. See Plaintiff’s Complaint attached hereto as “Exhibit A.”

2. Plaintiff’s complaint admits that the project for which the services were allegedly performed was

a single family residence. See Exhibit A at paragraph 12.

3. Plaintiff’s complaint avers that this is the first and only project Defendant is working, and has

worked, on. See Exhibit A at paragraph 13.

Preliminary Objection One

Based Upon

Legal Insufficiency of Defendant’s Pleading

Under the Contractor and Subcontractor’s Payment Act

4. Defendant incorporates all averments of these preliminary objections herein by reference as if

more fully set forth at length.

5. Count IV of Plaintiff’s complaint seeks to recover against Defendant under the Contractor and

Subcontractor Payment Act, 73 P.S. § 501, et seq.

6. The Contractor and Subcontractor Payment Act is inapplicable to the instant matter.

7. The Contractor and Subcontractor Payment Act states "[t]his Act shall not apply to

improvements to real property which consists of six or fewer residential units which are under construction

simultaneously." 73 P.S. § 503(a).

8. Defendant owns only one property, which is not residential.

9. Defendant did not have six or more residential units under construction simultaneously.

10. Therefore, the Contractor and Subcontractor Payment Act is inapplicable to the instant matter,

rendering Plaintiff’s complaint legally insufficient.

WHEREFORE, Defendant respectfully requests that the complaint be dismissed for legal insufficiency

of a pleading for failure to state a claim for which relief can be granted pursuant to Pennsylvania Rule of Civil

Procedure 1028(a)(4).

Legal Argument

[Draft legal argument here.]

66

Page 71: JULY 2018 PENNSYLVANIA BAR EXAMINATION · JULY 2018 PENNSYLVANIA BAR EXAMINATION . Essay Questions and Examiners’ Analyses. and . Performance Test . Pennsylvania Board of Law Examiners

LIBRARY

67

Page 72: JULY 2018 PENNSYLVANIA BAR EXAMINATION · JULY 2018 PENNSYLVANIA BAR EXAMINATION . Essay Questions and Examiners’ Analyses. and . Performance Test . Pennsylvania Board of Law Examiners

592 A.2d 716 (Pa.Super. 1991)

Howard W.F. WOLFE, Jr., Appellant,

v.

Joseph B. PORTER, Appellee.

* * *

The dispute in this action of ejectment is between neighbors and pertains to the ownership of a seventeen (17)

foot wide strip of land along a common property line. Joseph B. Porter, the defendant, holds record title to the

disputed land. Howard W.F. Wolfe, Jr., the plaintiff, claims title by adverse possession. In order to establish his

claim, however, it is necessary that Wolfe tack his possession to a period in which his parents allegedly possessed

the land in dispute. The trial court held that tacking, under the facts alleged in plaintiff's complaint, was not

available to establish title by adverse possession and sustained defendant's preliminary objections in the nature of

a demurrer to plaintiff's amended complaint. After careful review, we affirm. [footnote omitted]

The standard of [trial and] appellate review was stated in Ward v. Serfas, [citation omitted] as follows:

When preliminary objections in the nature of a demurrer are filed, we must accept as true all the

well-pleaded material facts set forth in the complaint and all reasonable inferences deducible from

those facts. Accepting these facts and inferences, we then determine whether the pleader has failed

to state a claim for which relief may be granted, and we will affirm the grant of a demurrer only if

there is certainty that no recovery is possible. All doubts are resolved in favor of the pleader.

Furthermore, by filing preliminary objections in the nature of a demurrer, appellees have admitted

the factual allegations of the complaint for purposes of the demurrer.

[citations omitted].

* * *

A review of the facts alleged in the complaint . . . makes it apparent that plaintiff cannot establish title to the

disputed land by adverse possession.

"[O]ne who claims title by adverse possession must prove that he had actual, continuous, exclusive, visible,

notorious, distinct, and hostile possession of the land for twenty-one years." [citation omitted]. In the instant

case, the plaintiff-appellant has been in possession of the disputed tract of land since 1979. This is an insufficient

period to establish title by adverse possession.

Appellant argues, however, that when the period of his possession is tacked on to the period during which his

predecessor in title held possession, a continuous and adverse possession for more than 21 years is established.

The applicable law was stated in Wittig v. Carlacci, [citation omitted] as follows:

* * *

The possession of successive occupants may be tacked, but only where there is privity between

them. For our purposes, "privity" refers to a succession of relationship to the same thing,

68

Page 73: JULY 2018 PENNSYLVANIA BAR EXAMINATION · JULY 2018 PENNSYLVANIA BAR EXAMINATION . Essay Questions and Examiners’ Analyses. and . Performance Test . Pennsylvania Board of Law Examiners

whether created by deed or other acts or by operation of law.

But a deed does not of itself create privity between the grantor and the grantee as to land not

described in the deed but occupied by the grantor in connection therewith, although the grantee

enters into possession of the land not described and uses it in connection with that conveyed * *

* The deed, in itself, creates no privity as to land outside its calls. Nor is privity created by the

bare taking of possession of land previously occupied by the grantor.

Our court has held that acceptance of a deed describing boundary lines confined the premises to

the area within the boundaries, and that such a deed did not convey inchoate [meaning: “an

interest in real estate which is not a present interest, but which may ripen into a vested estate, if

not barred, extinguished, or divested.” Black’s Law Dictionary, Sixth Ed. (1990)] rights

acquired by incompleted adverse possession. [citation omitted] Each predecessor must have

claimed title to the property in dispute, and in transferring to his successors must have purported

to include it.

[citations and footnote omitted]

Thus, a grantee cannot tack his grantor's possession of land when the grantor does not convey such

land to him. The only recognized exception occurs where an intent to convey more land than that

described may be inferred from the circumstances or the deed itself.

[citations omitted]

Plaintiff-appellant's title to his land was acquired by deed from his mother, Catherine M. Conrad, formerly

Catherine M. Wolfe, on July 31, 1979. Prior thereto, the land had been owned by plaintiff's parents, Howard

W.F. Wolfe and Catherine M. Wolfe, husband and wife, who had acquired title by deed in 1952. Howard W.F.

Wolfe died April 27, 1978; whereupon, Catherine M. Wolfe became the sole owner thereof. Plaintiff's parents,

it is alleged, were in continuous possession of their own land and also of the disputed tract between 1952 and

1979. There is no suggestion, however, that title by adverse possession was perfected by adjudication or, indeed,

that a claim of title by adverse possession was ever made against the owners of the adjoining tract.

When Catherine M. Wolfe (Conrad) conveyed to her son, she conveyed only the land for which she had record

title. The deed contained a metes and bounds description of the land; and this description did not include any part

of the land now in dispute. Where title by adverse possession is inchoate, a deed by a grantor which fails to convey

such inchoate right is ineffective to create privity which allows tacking. The grantor's failure to include the

disputed tract in the 1979 conveyance to her son prevented the son from tacking his possession of the disputed

tract on to that of his parents to establish his own title by adverse possession. [citation omitted]

Appellant argues that an intent to include the disputed tract can be inferred from the language of the deed. We

disagree. A careful examination of the deed discloses no reference to the disputed tract or to any inchoate right

thereto. "The deed, in itself, creates no privity as to land outside its calls." [citation omitted] In the absence of a

conveyance of the grantor's inchoate rights, we assume that appellant's predecessors either occupied the disputed

land permissively or that they abandoned every intention of holding adversely before the conveyance. [citations

omitted] (where grantor had claim of adverse possession to certain tract of land, and subsequent deed to grantee

specifically excluded that tract, grantee was precluded from asserting continuing claim of adverse possession,

69

Page 74: JULY 2018 PENNSYLVANIA BAR EXAMINATION · JULY 2018 PENNSYLVANIA BAR EXAMINATION . Essay Questions and Examiners’ Analyses. and . Performance Test . Pennsylvania Board of Law Examiners

since he cannot be said to be in privity with grantor); [citation omitted] (although party claiming title by adverse

possession may tack possession of predecessors, party must show sufficient privity between them and that each

of them has transferred to his successor in some lawful manner the adverse possession or inchoate interest held).

* * *

As we have observed, appellant's own possession falls short of the required twenty-one (21) year period and the

amended complaint discloses that there is no privity between appellant and his predecessors in title which would

enable him to tack his period of possession to the possession of his predecessors. Therefore, appellant has failed

to state a claim for which relief may be granted, and the trial court properly granted appellee's preliminary

objection in the nature of a demurrer.

Affirmed.

70

Page 75: JULY 2018 PENNSYLVANIA BAR EXAMINATION · JULY 2018 PENNSYLVANIA BAR EXAMINATION . Essay Questions and Examiners’ Analyses. and . Performance Test . Pennsylvania Board of Law Examiners

812 A.2d 558 (Pa. 2002)

Frank A. ZEGLIN, Jr., and Tammy Lee Zeglin, Appellants,

v.

Sean E. GAHAGEN and Kimberlee H. Gahagen, Appellees.

* * *

In this appeal involving a boundary dispute, the question presented is whether privity of estate between succeeding

landowners is required to support tacking periods of ownership to form the requisite twenty-one-year period under

acquiescence theory.

Appellants, Frank and Tammy Zeglin, and Appellees, Sean and Kimberlee Gahagen, own adjoining properties in

Windber, Paint Township, Somerset County. The Zeglins purchased in 1977 from Cora Murphy, who, together

with her late husband, had owned the property since 1937. The Gahagens bought from Margaret Swincinski in

1989, who had acquired the parcel in 1979 from the previous owners since 1972.

In 1995, the Gahagens employed a professional to survey their property and learned that their deed described a

boundary on the Zeglins' side of a line marked by a row of bushes, utility pole, and fence that had been added by

the Zeglins. The surveyor therefore concluded that the Gahagens' property extended over such visible line, and

this was confirmed in a subsequent survey commissioned by the Zeglins. The Gahagens notified the Zeglins that

a portion of their driveway encroached on their land, removed the bushes, and constructed a retaining wall

adjacent to the surveyed boundary. The Zeglins responded by filing a complaint against the Gahagens sounding

in ejectment and trespass and claiming ownership up to the line previously demarcated by the bushes, utility pole,

and fence. In furtherance of this position, the Zeglins relied, inter alia, on the doctrine of acquiescence in a

boundary, alleging that their occupancy and possession, together with that of their predecessors in title, for a

period of more than twenty-one years established the visible line as the legal boundary. The Gahagens filed an

answer and counterclaim.

In March of 2000, following a non-jury trial, the common pleas court issued a decree nisi in favor of the Zeglins,

which it later made final. In accompanying opinions, the court summarized the acquiescence doctrine as follows:

an occupation up to a fence on each side by a party or two parties for more than 21 years, each

party claiming the land on his side as his own, gives to each an incontestable right up to the fence,

whether the fence is precisely on the right line or not; and this is so although the parties may not

have consented specifically to the fence in question.

[citations omitted]. The court identified as the basis for the principle public policy favoring peace and the repose

of titles. It reasoned that, for a period of more than twenty-one years, the Zeglins, the Gahagens, and their

predecessors in interest had recognized and acquiesced in a boundary line demarcated by the hedgerow (and also

highlighted by the fence maintained by the Zeglins through a portion of that time period). Although the Zeglins

had occupied the property for only eighteen years prior to the Gahagens' actions, the court permitted them to tack

the period of ownership by the Murphys, despite the fact that Cora Murphy had not specifically and formally

71

Page 76: JULY 2018 PENNSYLVANIA BAR EXAMINATION · JULY 2018 PENNSYLVANIA BAR EXAMINATION . Essay Questions and Examiners’ Analyses. and . Performance Test . Pennsylvania Board of Law Examiners

conveyed her purported interest in the disputed tract to the Zeglins in the written deed. As pertains to tacking

under the doctrine of adverse possession, the court recognized the requirement in Pennsylvania of privity of estate,

namely, a higher degree of relation than that of mere grantor and grantee of a main parcel, generally comprised

of specific and formal conveyance of the predecessor's interest in the disputed tract where the transfer is between

unrelated parties. [citations omitted] The common pleas court found, however, that Pennsylvania courts had

distinguished acquiescence in a boundary by applying the less rigorous requirement of privity of possession to

claims predicated on such theory. [citation omitted] ("Pennsylvania courts have adopted the view that succeeding

owners of property are bound by the fences that were accepted and recognized by former owners even without

any other privity or formal transfer of the area possessed adversely." [citation omitted]).

On the Gahagens' appeal, the Superior Court reversed . . . . The court determined, however, that, just as in the

case of adverse possession, privity of estate is an essential prerequisite to employment of tacking to perfect a

claim under acquiescence theory. [citations omitted] Accordingly, the Superior Court held that the common

pleas court erred by permitting the Zeglins to tack the period of the Murphys' ownership based on privity of

possession alone. [citation omitted]

Presently, the Zeglins argue that privity of estate as a prerequisite to tacking is inappropriate to, and contrary to

the doctrine of, acquiescence in a boundary, since an underlying premise of such theory is that the evidence of

longstanding acquiescence in a physical boundary by adjoining property owners will control over contrary deed

calls. The Zeglins distinguish Plott v. Cole, cited by the Superior Court, as allowing for creation of privity by

"other acts," and not solely by references culled from a deed. The Gahagens concede that the privity of estate

requirement has not expressly been attached by Pennsylvania courts in acquiescence cases, but contend that such

a requirement would alleviate confusion among landowners.

The establishment of a boundary line by acquiescence for the statutory period of twenty-one years has long been

recognized in Pennsylvania. [1] Two elements are prerequisites: 1) each party must have claimed and occupied

the land on his side of the line as his own; and 2) such occupation must have continued for the statutory period of

twenty-one years. [citation omitted] As recognized by the Superior Court and the common pleas court, the

doctrine functions as a rule of repose to quiet title and discourage vexatious litigation. See id. at 592, 547 A.2d

at 1220.

* * *

As President Judge Coffroth [of the Somerset County Court of Common Pleas] aptly observed, the reason why

privity of estate should not be deemed necessary to support tacking in this setting is, simply, because a prospective

purchaser will see the fence or similar marking; given its "obvious presence as apparent boundary," he is therefore

put on notice to inquire about its origin, history, and function. [citation omitted] [5] [12] ("After 21 years, the

chips will be allowed to fall where they may, for reasons of equity and peace.").

Accordingly, we find the majority view (requiring only privity of possession) better suited to claims brought

under a theory of acquiescence in a boundary. We hold, therefore, that tacking is permitted in such context upon

sufficient and credible proof of delivery of possession of land not within (but contiguous to) property described

72

Page 77: JULY 2018 PENNSYLVANIA BAR EXAMINATION · JULY 2018 PENNSYLVANIA BAR EXAMINATION . Essay Questions and Examiners’ Analyses. and . Performance Test . Pennsylvania Board of Law Examiners

by deed of conveyance, which was previously claimed and occupied by the grantor and is taken by the grantee as

successor in such interest.

The order of the Superior Court is reversed, and the case is remanded. . . .

* * *

Notes:

[1] See Reiter v. McJunkin, 173 Pa. 82, 84, 33 A. 1012 (1896) ("After 21 years of occupancy up to a fence on

each side as a line fence, it is not material to inquire whether the fence is on the right line or not."); [citations

omitted] ("a boundary line may be proved by a long-standing fence without proof of a dispute and its settlement

or compromise") [citation omitted].

* * *

[5] "[A]cquiescence" in the context of disputed boundaries "denotes passive conduct on the part of the lawful

owner consisting of failure on his part to assert his paramount rights or interests against the hostile claims of the

adverse user." [citation omitted] . . . .

* * *

[12] [citations omitted] (indicating that "color of title [is] not necessary for tacking to provide continuity of

possession of land, provided the land in question [is] contiguous to that described in a deed, and that lands both

titled and untitled were part of a close, apparent by reason of physical boundaries such as fences or hedges."

[citations omitted]

73

Page 78: JULY 2018 PENNSYLVANIA BAR EXAMINATION · JULY 2018 PENNSYLVANIA BAR EXAMINATION . Essay Questions and Examiners’ Analyses. and . Performance Test . Pennsylvania Board of Law Examiners

Instructions

The performance test is designed to test an applicant’s ability to perform the legal

task that has been assigned using the factual information contained in the File and

legal principles that are provided in the Library.

The File contains the only factual information that you should consider in

performing the assigned task. The task to be completed is set forth in the first

document in the File in the form of a memorandum to the applicant. The Library

contains the only legal principles that you should consider to complete the assigned

task. Although your general knowledge of the law may provide some background

for analyzing the problem, the factual information contained in the File and the

legal principles contained in the Library are the only materials that you should use

in formulating your answer to the assigned task.

Your response should be written in the gray answer book or typed in answer screen

number 3 of SofTest. Be sure to allow sufficient time for reading the materials,

organizing your answer and completing the task assigned. Your answer should

demonstrate an understanding of the relevant facts, recognition of the issues and the

applicable principles of law and the reasoning that supports your answer. Your

grade will be based on the content of your response and your ability to follow

instructions in performing the assigned task.

The events depicted and the persons portrayed by the information in the File are

fictitious and such information does not depict nor is it intended to depict or portray

any actual person, company or occurrence. Any similarity to any person, living or

dead, or any occurrence is purely coincidental.

74

Page 79: JULY 2018 PENNSYLVANIA BAR EXAMINATION · JULY 2018 PENNSYLVANIA BAR EXAMINATION . Essay Questions and Examiners’ Analyses. and . Performance Test . Pennsylvania Board of Law Examiners

Question No. PT: Examiner’s Analysis

The applicant is assigned to draft preliminary objections to a complaint that seeks to quiet title via adverse

possession and consentable/acquiescence in a boundary; and to draft the legal argument section of a brief in

support of those preliminary objections. Applicants are instructed to limit their preliminary objections to a

single demurrer to each of the two counts of the complaint. The caselaw provided should lead applicants to

focus their demurrers on: 1. The inability of the plaintiff to “tack” his claim onto his predecessor’s claim to

meet the 21 year time limit under the theory of adverse possession; and 2. The concept of

consentable/acquiescence in a boundary is inappropriate because our case does not involve a boundary dispute,

as there is no identifiable boundary line in question.

Drafting the Preliminary Objections 3 Points

The applicant should follow the directions in the assignment memorandum and formatting memorandum and, in

formatting the preliminary objections, should include separately numbering paragraphs, address each objection

in a separate section, and draft both preliminary objections and a legal argument section of a brief.

The applicant should identify the following two preliminary objections, and set forth the basis for those

objections:

1. Plaintiff’s complaint is legally insufficient because it fails to state a valid cause of action for

adverse possession where the Plaintiff has not possessed the tract in question for the required 21

years and cannot “tack” on his predecessor’s purported use;

2. Plaintiff’s complaint is legally insufficient because it fails to set forth all elements of

acquiescence in a boundary, specifically that there exists no actual boundary.

The applicant should also address the two preliminary objections in a legal argument section that the managing

partner will include in his brief in support of preliminary objections. The analysis for the preliminary objections

and legal argument should be as follows:

Plaintiff’s complaint is legally insufficient because it fails to state a valid cause of action for adverse

possession because he has not possessed the tract in question for 21 years and he cannot “tack” on

purported use of the tract in question by his predecessor in interest. 9 Points

Pennsylvania Rule of Civil Procedure 1028(a)(4) permits preliminary objections to be filed to any pleading on

the basis of legal insufficiency (demurrer). Formatting Memorandum. The trial court’s standard of review of a preliminary objection in the nature of a demurrer is as follows:

When preliminary objections in the nature of a demurrer are filed, we must accept as true all the

well-pleaded material facts set forth in the complaint and all reasonable inferences deducible

from those facts. Accepting these facts and inferences, we then determine whether the pleader

has failed to state a claim for which relief may be granted, and we will affirm the grant of a

demurrer only if there is certainty that no recovery is possible. All doubts are resolved in favor

of the pleader. Furthermore, by filing preliminary objections in the nature of a demurrer,

appellees have admitted the factual allegations of the complaint for purposes of the demurrer.

75

Page 80: JULY 2018 PENNSYLVANIA BAR EXAMINATION · JULY 2018 PENNSYLVANIA BAR EXAMINATION . Essay Questions and Examiners’ Analyses. and . Performance Test . Pennsylvania Board of Law Examiners

Wolfe v. Porter.

“One who claims title by Adverse Possession must prove that he had actual, continuous, exclusive, visible,

notorious, distinct, and hostile possession of the land for 21 years.” Wolfe v. Porter.

Plaintiff cannot meet all elements of adverse possession for the requisite 21 year period. In order to do so,

Plaintiff would have to have possessed the tract in question for twenty one years before he could successfully

assert a claim for adverse possession.

Plaintiff acquired Lot 3 in the Lake Anne Subdivision on February 1, 2008. Complaint at Exhibit A.

Because Plaintiff has only owned Lot 3 for a little more than ten (10) years, he would have to rely on the legal

concept of “tacking” in order to arrive at the required 21 year time period for adverse possession to be

recognized. Complaint and Wolfe v. Porter.

In order for tacking to apply, “[e]ach predecessor must have claimed title to the property in dispute, and in

transferring to his successors must have purported to include it.” Wolfe v. Porter.

“Thus a grantee cannot tack his grantor’s possession of land when the grantor does not convey such land to

him.” Wolfe v. Porter.

When a deed of conveyance contains a metes and bounds description of the land conveyed and said description

does not include the land in dispute tacking cannot be established in an adverse possession claim. Wolfe v. Porter.

The deed attached to plaintiff’s complaint at Exhibit A for Lot 3 contains a metes and bounds description for

Lot 3, and transfers title to only Lot 3 from an Ethan Bishop to Plaintiff, Paul Kerrigan. Complaint at Exhibit

A.

Nowhere in plaintiff’s deed is there mention of the portion of defendant’s property that is across the public dirt

road from Lot 3; nor does the conveyance make any reference to the tract of land over which Plaintiff claims

adverse possession.

To the contrary, the tract in question is a part of the property deeded to defendant. Complaint at Exhibit B.

In a claim for adverse possession “[t]he possession of successive occupants may be tacked, but only where there

is privity between them.” Wolfe v. Porter.

“Privity refers to a succession of relationship to the same thing, whether created by deed or other acts or by

operation of law . . . the deed, in itself, creates no privity as to land outside its calls. Nor is privity created by

the bare taking of possession of land previously occupied by the grantor.” Wolfe v. Porter.

A deed does not of itself create privity between the grantor and the grantee as to land not described in

the deed but occupied by the grantor in connection therewith, although the grantee enters into possession

of the land not described and uses it in connection with that conveyed * * * The deed, in itself, creates

no privity as to land outside its calls. Nor is privity created by the bare taking of possession of land

previously occupied by the grantor. Wolfe v. Porter.

76

Page 81: JULY 2018 PENNSYLVANIA BAR EXAMINATION · JULY 2018 PENNSYLVANIA BAR EXAMINATION . Essay Questions and Examiners’ Analyses. and . Performance Test . Pennsylvania Board of Law Examiners

Rather acceptance of a deed describing boundary lines confines the premises conveyed to the area within the

boundaries, and such a deed does not convey inchoate rights acquired by incompleted adverse possession. Wolfe v. Porter.

The absence of any mention, or even a hint of reference, to Lot 3 in Plaintiff’s Deed demonstrates that the

predecessor in title to Lot 3 did not purport to include the tract in question in the transfer to Plaintiff. Therefore,

there is no privity between the successors in title to the parcel across from Lot 3.

Plaintiff is not in privity with prior owners in the chain of title because the deed conveying his property rights

from 2008 to present made no reference to the tract across the road from Lot 3. Therefore, he cannot “tack” on

any purported use or claim of possession by prior owners. Thus, Plaintiff has failed to demonstrate the

necessary elements of adverse possession existed for at least 21 years, and his Complaint must be dismissed.

Plaintiff’s complaint is legally insufficient because it fails to state a valid cause of action in that it has

failed to demonstrate all elements of acquiescence in a boundary. 8 Points

Plaintiff’s complaint attempts to acquire ownership of the 40’ x 50’ tract directly across the public dirt road

from Lot 3, and which is a part of Lot 6, by “consentable/acquiescence in a boundary.” Complaint.

Acquiescence is a legal concept that addresses boundary line disputes between adjoining landowners. See generally, Zeglin v. Gahagen.

“The establishment of a boundary line by acquiescence for the statutory period of twenty-one years has long

been recognized in Pennsylvania. Two elements are prerequisites: 1) each party must have claimed and

occupied the land on his side of the line as his own; and 2) such occupation must have continued for the

statutory period of twenty-one years.” Zeglin v. Gahagen. Plaintiff has failed to identify the essential factor in an acquiescence claim: a disputed boundary line.

Plaintiff has failed to identify any mark, such as a tree line, fence, hedges, etc., that constitutes a boundary line

of the tract across the public dirt road from Lot 3. To the contrary, plaintiff’s complaint indicates that

defendant’s lot (Lot 6, containing the tract in question) is a large open field, with no fencing, trees or other

distinguishing characteristics. Complaint, paragraph 6. “[C]olor of title [is] not necessary for tacking to provide continuity of possession of land, provided the land in

question [is] contiguous to that described in a deed, and that lands both titled and untitled were part of a close,

apparent by reason of physical boundaries such as fences or hedges.” Zeglin v. Gahagen. A boundary line may be proved by a long-standing fence without proof of a dispute and its settlement or

compromise. Zeglin v. Gahagen. Because a prospective purchaser will see a fence or similar marking, given its obvious presence as apparent

boundary, he is . . . put notice to inquire about its origin, history, and function.” Zeglin v. Gahagen.

There simply is no boundary line presented by plaintiff upon which a consentable/acquiescence in a boundary

claim could be brought. The only boundary line identified in the complaint is the public dirt road between Lot 3

and Lot 6, and that is not the disputed line. Rather, plaintiff claims land on the defendant’s side of the public

dirt road, not on his own side of that boundary line, making a claim for acquiescence in a boundary improper.

77

Page 82: JULY 2018 PENNSYLVANIA BAR EXAMINATION · JULY 2018 PENNSYLVANIA BAR EXAMINATION . Essay Questions and Examiners’ Analyses. and . Performance Test . Pennsylvania Board of Law Examiners

Further, “‘acquiescence’ in the context of disputed boundaries ‘denotes passive conduct on the part of the

lawful owner consisting of failure on his part to assert his paramount rights or interests against the hostile

claims of the adverse user.’” Zeglin v. Gahagen.

Plaintiff’s complaint gives no indication that defendant has failed to assert his paramount rights or interests

against plaintiff’s claim, other than an implication that defendant has not objected to plaintiff’s use.

Because plaintiff has not established that he or his predecessors in title established any form of boundary lines

to distinguish the tract in question throughout the past 21 years, he cannot claim that the defendant acquiesced

to his possession of the tract. Thus, he has failed to demonstrate the necessary elements of acquiescence in a

boundary, and his complaint must be dismissed.

Furthermore, Pennsylvania caselaw dealing with the concept of acquiescence in a boundary generally addresses

situations in which a purported boundary has been established and agreed upon by the parties, and one party

claims ownership of the property up to the boundary line. Zeglin v. Gahagen. Here, as set forth above, the only

conceivable boundary line in question is the public dirt road. However, plaintiff is not simply claiming

ownership of the land up to the road; he is claiming ownership of the land for fifty (50) feet beyond the

boundary line. Thus, acquiescence in a boundary cannot apply in this instance, and plaintiff’s claim should be

dismissed.

78